Você está na página 1de 53

Stonehill vs.

Diokno [20 SCRA 383; L-19550; 19 June 1967]


Stonehill et al and the corporation they form were alleged to have committed acts in
violation of Central Bank Laws, Tariff and Customs Laws, Internal Revenue (Code)
and Revised Penal Code. By the strength of this allegation a search warrant was issued
against their persons and their corporation. The warrant provides authority to search the
persons above-named and/or the premises of their offices, warehouses and/or residences,
and to seize and take possession of the following personal property to wit:
Books of accounts, financial records, vouchers, correspondence, receipts, ledgers,
journals, portfolios, credit journals, typewriters, and other documents and/or papers
showing all business transactions including disbursements receipts, balance sheets and
profit and loss statements and Bobbins (cigarette wrappers).
The documents, papers, and things seized under the alleged authority of the warrants in
question may be split into (2) major groups, namely:
(a) those found and seized in the offices of the aforementioned corporations and
(b) those found seized in the residences of petitioners herein.
Stonehill averred that the warrant is illegal for:
(1) they do not describe with particularity the documents, books and things to be seized;
(2) cash money, not mentioned in the warrants, were actually seized;
(3) the warrants were issued to fish evidence against the aforementioned petitioners in
deportation cases filed against them;
(4) the searches and seizures were made in an illegal manner; and
(5) the documents, papers and cash money seized were not delivered to the courts that
issued the warrants, to be disposed of in accordance with law.
The prosecution counters, invoking the Moncado doctrine, that the defects of said
warrants, if any, were cured by petitioners consent; and (3) that, in any event, the
effects seized are admissible in evidence against them. In short, the criminal cannot be set
free just because the government blunders.
ISSUE: Whether or not the search warrant issued is valid.
HELD: The SC ruled in favor of Stonehill et al. The SC emphasized however that
Stonehill et al cannot assail the validity of the search warrant issued against their
corporation for Stonehill are not the proper party hence has no cause of action. It should
be raised by the officers or board members of the corporation. The constitution protects

the peoples right against unreasonable search and seizure. It provides; (1) that no
warrant shall issue but upon probable cause, to be determined by the judge in the manner
set forth in said provision; and (2) that the warrant shall particularly describe the things to
be seized. In the case at bar, none of these are met.
The warrant was issued from mere allegation that Stonehill et al committed a violation
of Central Bank Laws, Tariff and Customs Laws, Internal Revenue (Code) and Revised
Penal Code. In other words, no specific offense had been alleged in said applications.
The averments thereof with respect to the offense committed were abstract.
As a consequence, it was impossible for the judges who issued the warrants to have found
the existence of probable cause, for the same presupposes the introduction of competent
proof that the party against whom it is sought has performed particular acts, or committed
specific omissions, violating a given provision of our criminal laws. As a matter of fact,
the applications involved in this case do not allege any specific acts performed by herein
petitioners. It would be a legal heresy, of the highest order, to convict anybody of a
violation of Central Bank Laws, Tariff and Customs Laws, Internal Revenue (Code)
and Revised Penal Code, as alleged in the aforementioned applications without
reference to any determinate provision of said laws or codes.
The grave violation of the Constitution made in the application for the contested search
warrants was compounded by the description therein made of the effects to be searched
for and seized, to wit:
Books of accounts, financial records, vouchers, journals, correspondence, receipts,
ledgers, portfolios, credit journals, typewriters, and other documents and/or papers
showing all business transactions including disbursement receipts, balance sheets and
related profit and loss statements.
Thus, the warrants authorized the search for and seizure of records pertaining to all
business transactions of Stonehill et al, regardless of whether the transactions were legal
or illegal. The warrants sanctioned the seizure of all records of Stonehill et al and the
aforementioned corporations, whatever their nature, thus openly contravening the explicit
command of the Bill of Rights that the things to be seized be particularly described
as well as tending to defeat its major objective: the elimination of general warrants. The
Moncado doctrine is likewise abandoned and the right of the accused against a defective
search warrant is emphasized.

Soliven v Makasiar Nov 14, 1988 G.R. No. 82585


Facts: In these consolidated cases, three principal issues were raised: (1) whether or not
petitioners were denied due process when informations for libel were filed against them
although the finding of the existence of a prima facie case was still under review by the
Secretary of Justice and, subsequently, by the President; and (2) whether or not the
constitutional rights of Beltran were violated when respondent RTC judge issued a
warrant for his arrest without personally examining the complainant and the witnesses, if

any, to determine probable cause. Subsequent events have rendered the first issue moot
and academic. On March 30, 1988, the Secretary of Justice denied petitioners' motion for
reconsideration and upheld the resolution of the Undersecretary of Justice sustaining the
City Fiscal's finding of a prima facie case against petitioners. A second motion for
reconsideration filed by petitioner Beltran was denied by the Secretary of Justice on April
7, 1988. On appeal, the President, through theExecutive Secretary, affirmed the resolution
of the Secretary of Justice on May 2, 1988. The motion for reconsideration was denied by
theExecutive Secretary on May 16, 1988. With these developments, petitioners'
contention that they have been denied the administrative remedies available under the law
has lost factual support.
Issues:
(1) Whether or Not petitioners were denied due process when informations for libel were
filed against them although the finding of the existence of a prima facie case was still
under review by the Secretary of Justice and, subsequently, by the President.
(2) Whether or Not the constitutional rights of Beltran were violated when respondent
RTC judge issued a warrant for his arrest without personally examining the complainant
and the witnesses, if any, to determine probable cause
Held: With respect to petitioner Beltran, the allegation of denial of due process of law in
the preliminary investigation is negated by the fact that instead of submitting his counteraffidavits, he filed a "Motion to Declare Proceedings Closed," in effect waiving his right
to refute the complaint by filing counter-affidavits. Due process of law does not require
that the respondent in a criminal case actually file hiscounter-affidavits before the
preliminary investigation is deemed completed. All that is required is that the respondent
be given theopportunity to submit counter-affidavits if he is so minded.
The second issue, raised by petitioner Beltran, calls for an interpretation of the
constitutional provision on the issuance of warrants of arrest. The pertinent provision
reads:
Art. III, Sec. 2. The right of the people to be secure in their persons, houses, papers and
effects against unreasonable searches and seizures of whatever nature and for any purpose
shall be inviolable, and no search warrant or warrant of arrest shall issue except upon
probable cause to be determined personally by the judge afterexamination nder oath or
affirmation of the complainant and the witnesses he may produce, and particularly
describing the place to be searched and the persons or things to be seized.
The addition of the word "personally" after the word "determined" and the deletion of the
grant of authority by the 1973 Constitution to issue warrants to "other responsible officers
as may be authorized by law," has apparently convinced petitioner Beltran that the
Constitution now requires the judge to personally examine the complainant and his
witnesses in his determination of probable cause for the issuance of warrants of arrest.
This is not an accurate interpretation.

What the Constitution underscores is the exclusive and personal responsibility of the
issuing judge to satisfy himself of the existence of probable cause. In satisfying himself
of the existence of probable cause for the issuance of a warrant of arrest, the judge is not
required to personally examine the complainant and his witnesses. Following established
doctrine and procedure, he shall: (1) personally evaluate the report and the supporting
documents submitted by the fiscal regarding the existence of probable cause and, on the
basis thereof, issue a warrant of arrest; or (2) if on the basis thereof he finds no probable
cause, he may disregard the fiscal's report and require the submission of supporting
affidavits of witnesses to aid him in arriving at a conclusion as to the existence of
probable cause.
Sound policy dictates this procedure, otherwise judges would be unduly laden with the
preliminary examination and investigation of criminal complaints instead of
concentrating on hearing and deciding cases filed before their courts. It has not been
shown that respondent judge has deviated from the prescribed procedure. Thus, with
regard to the issuance of the warrants of arrest, a finding of grave abuse of discretion
amounting to lack or excess of jurisdiction cannot be sustained. The petitions fail to
establish that public respondents, through their separate acts, gravely abused their
discretion as to amount to lack of jurisdiction. Hence, the writs of certiorari and
prohibition prayed for cannot issue.
WHEREFORE, finding no grave abuse of discretion amounting to excess or lack of
jurisdiction on the part of the public respondents, the Court Resolved to DISMISS the
petitions in G. R. Nos. 82585, 82827 and 83979. The Order to maintain the status quo
contained in the Resolution of the Court en banc dated April 7, 1988 and reiterated in the
Resolution dated April 26, 1988 is LIFTED.

SILVA VS. PRESIDING JUDGE [203 SCRA 140; G.R. No. 81756; 21 Oct
1991]
Wednesday, February 04, 2009 Posted by Coffeeholic Writes
Labels: Case Digests, Political Law
Facts: Sgt. Villamor, chief of the PC Narcom Detachment in Dumaguete City filed an
"application for search warrant" and "Deposition of witness" against petitioner
Nicomedes Silva and Martin Silva. Judge Nickarter Ontal, then the presiding judge of
RTC of Dumaguete issued Search Warrant No.1 pursuant to the said applications for
violation of RA 6425 Dangerous DrugsACT of 1972. Such warrant states that there is a
probable cause to believe that Mr. Tama Silva has the possession and control of marijuana
dried leaves, cigarette and joint. The warrant authorizes Sgt. Villamor to make an
immediate search at any time of the room of Mr. Tama Silva at the residence of his father
Comedes Silva and to open aparadors, lockers, cabinets, cartons and containers to look
for said illegal drugs. In the course of the search, the officers seized money belonging to

Antonieta Silva in the amount of P1,231.40. Petitioner filed a motion to quash Search
WarrantNo.1 on the ground that 1) it was issued on the sole basis of mimeographed 2)
the judge failed to personally examine the complainant and witness by
searching questions and answers.

Issue: Whether or Not Search Warrant No.1 is invalid. WON the officers abused their
authority in seizing the money of Antonieta Silva.

Held: Search Warrant No. 1 is invalid due to the failure of the judge to examine
the witness in the form of searching questions and answers. The questions asked were
leading as they are answerable by mere yes or no. Such questions are not sufficiently
searching to establish probable cause. The questions were already mimeographed and all
the witness had to do was fill in their answers on the blanks provided. Judge Ontal is
guilty of grave abuse of discretion when he rejected the motion of Antonieta Silvaseeking
the return of her money.
The officers who implemented the search warrant clearly abused their authority when
they seized the money of Antonieta Silva. The warrant did not indicate the seizure of
money but only for marijuana leaves, cigarettes..etc. Search Warrant No. 1 is declared
null and void.
*** Sec 4 Rule 126 Rules of Court
Examination of the complainant, record -the judge before issuing thewarrant, personally
examine in the form of searching questions and answers, in writing and under oath the
complainant and any witness he may produce the facts personally known to them and
attach to the record their sworn statements together with their affidavits.

NICOMEDES SILVA vs. THE HONORABLE PRESIDING JUDGE


G.R. No. 81756

October 21, 1991

Facts:
M/Sgt. Ranulfo Villamor, as chief of the PC NARCOM Detachment in
Dumaguete City, Negros Oriental filed an application for the search warrant with the
RTC against petitioners. The application was accompanied by deposition of witness
executed by Arthur Alcoran and Pat. Leon Quindo.
Judge Hickarter Ontal, Presiding judge issued search warrant no. 1 directing the
aforesaid police officers to search the room of Marlon Silva in the residence of
Nicomedes Silva for violation of the dangerous drugs law.. under the search warrant its
state that :seize and take possession of the following property marijuana, dried leaves,
cigarettes, joint and bring said property to the undersigned to be dealt with as the law
directs.
In the course of the search, the serving officer also seized money belonging to
Antoinette Silva in the amount of 1231.40. Antoinette filed a motion the return of the said
amount. Acting on said motion Judge Ontal issued an order stating that the court holds
in abeyance the disposition of the said amount pending the filing of appropriate charges
in connection with the search warrant.
Issue:
Whether or not there is a violation of the constitutional right against unreasonable
search and seizure
Ruling:
The Supreme Court held that Section 3 and 4, Rule 126 of the Rules of Court
provides for the requisite for the issuance oa a search warrant.
Section 3 a search warrant shall not issue except for probable cause in connection with
one specific offense to be determined personally by the Judge after eamination under oath

Morano
Facts: Petitioners are Chinese nationals (Chan Sau Wah from Fukein, with a minor child
from prior marriage, Fu Yan Fun) who were granted a temporary visitors visa as momimmigrant for 2 months upon posting P4k cash bond to visit a cousin in the Philippines.
She soon married to Esteban Morano, a Filipino Citizen, on January 24, 1962 and gave
birth to a child, Esteban Morano, Jr. After several extensions to prolong stay in
Philippines, their visas expired on Sept. 10, 1962 and were ordered by Commissioner of
Immigration (COI) on Aug. 31, 1962 thru a letter, to leave the country on or before Sept.
10, 1962 with warning of issuance of warrant of arrest for failure to leave and
confiscation of bond.
Petitioners then filed with the CFI of Manila for Mandamus to compel COI to cancel their
ACR, to stop issuing arrest warrant, and preliminary injunction from confiscating their

bond. They argue that Chan Sau Wah became a Filipino Citizen upon marriage to Esteban
Morano by virtue of Section 15 of Commonwealth Act No. 473 (Revised Naturalization
Act). Likewise, it argues that Section 37 of the Naturalization Law is unconstitutional for
allowing the COI to issue warrant of arrest and effecting deportation without judicial
intervention enshrined in the Constitution. CFI decided partly against petitioners, thus,
COI and petitioners both appealed to SC.
Issues:
Whether or not the marriage of Chan Sau Wah to Esteban Morano makes her a Filipino
citizen.
Whether or not Section 37 of the Naturalization Law empowering the COI to issue a
warrant of arrest, and deport upon a warrant on deportation cases is unconstitutional for
are covered by the Constitutional mandate on searches and seizures without judicial
intervention required under the Constitution on searches and seizures.
Ruling:
Searches and seizures. Power to deport aliens is an attribute of sovereignty planted on
the accepted maxim of international law, that every sovereign nation has the power, as
inherent in sovereignty, and essential to self-preservation, to forbid the entrance of
foreigners within its dominions. Section 1 (3), Article III of the Constitution, does not
require judicial intervention in the execution of a final order of deportation issued in
accordance with law. The constitutional limitation contemplates an order of arrest in the
exercise of judicial power as a step preliminary or incidental to prosecution or proceeding
for a given offense or administrative action, not as a measure indispensable to carry out a
valid decision by a competent official, such as a legal order of deportation, issued by the
Commissioner of Immigration, in pursuance of a valid legislation.
Petition for mandamus and prohibition with respect to petitioners Chan Sau Wah is
hereby denied; and judgment declaring her a citizen of the Philippines, directing COI to
cancel her Alien Certificate of Registration and other immigration papers, and declaring
the preliminary injunction with respect to her permanent, are all hereby set aside; and in
all other respects, the decision appealed is hereby affirmed.

HARVEY V. DEFENSOR-SANTIAGO [162 SCRA 840; G.R. NO. 82544; 28


JUN 1988]
Wednesday, February 04, 2009 Posted by Coffeeholic Writes
Labels: Case Digests, Political Law
F acts: This is a petition for Habeas Corpus. Petitioners are the following:American
nationals Andrew Harvey, 52 and Jonh Sherman 72. Dutch Citizen Adriaan Van Den
Elshout, 58. All reside at Pagsanjan Laguna respondent Commissioner Miriam Defensor
Santiago issued Mission Orders to the Commission of Immigration and Deportation
(CID) to apprehended petitioners at their residences. The Operation Report read that
Andrew Harvey was found together with two young boys. Richard Sherman was found
with two naked boys inside his room. While Van Den Elshout in the after Mission

Report read that two children of ages 14 and 16 has been under his care and subjects
confirmed being live-in for sometime now.
Seized during the petitioners apprehension were rolls of photo negatives and photos of
suspected child prostitutes shown in scandalous poses as well as boys and girls engaged
in sex. Posters and other literature advertising the child prostitutes were also found.
Petitioners were among the 22 suspected alien pedophiles. They were apprehended 17
February1988 after close surveillance for 3 month of the CID in Pagsanjan, Laguna. 17 of
the arrested aliens opted for self-deportation. One released for lack of evidence, another
charged not for pedophile but working with NO VISA, the 3 petitioners chose to face
deportation proceedings. On 4 March1988, deportation proceedings were instituted
against aliens for being undesirable aliens under Sec.69 of Revised Administrative Code.
Warrants of Arrest were issued 7March1988 against petitioners for violation of Sec37, 45
and 46 of Immigration Act and sec69 of Revised Administrative Code. Trial by the Board
of Special Inquiry III commenced the same date. Petition for bail was filed 11March 1988
but was not granted by the Commissioner of Immigration. 4 April1988 Petitioners filed a
petition for Writ of Habeas Corpus. The court heard the case on oral argument on 20
April 1988.

Issues:
(1) Whether or Not the Commissioner has the power to arrest and detain petitioners
pending determination of existence of probable cause.
(2) Whether or Not there was unreasonable searches and seizures by CIDagents.
(3) Whether or Not the writ of Habeas Corpus may be granted to petitioners.

Held: While pedophilia is not a crime under the Revised Penal Code, it violates the
declared policy of the state to promote and protect the physical, moral, spiritual and
social well being of the youth. The arrest of petitioners was based on the probable cause
determined after close surveillance of 3 months. The existence of probable cause justified
the arrest and seizure of articles linked to the offense. The articles were seized as an
incident to a lawful arrest; therefore the articles are admissible evidences (Rule 126,

Section12 of Rules on Criminal Procedure).


The rule that search and seizures must be supported by a valid warrant of arrest is not an
absolute rule. There are at least three exceptions to this rule. 1.) Search is incidental to the
arrest. 2.) Search in a moving vehicle. 3.) Seizure of evidence in plain view. In view of
the foregoing, the search done was incidental to the arrest.
The filing of the petitioners for bail is considered as a waiver of any irregularity attending
their arrest and estops them from questioning its validity. Furthermore, the deportation
charges and the hearing presently conducted by the Board of Special Inquiry made their
detention legal. It is a fundamental rule that habeas corpus will not be granted when
confinement is or has become legal, although such confinement was illegal at the
beginning.
The deportation charges instituted by the Commissioner of Immigration are in accordance
with Sec37 (a) of the Philippine Immigration Act of 1940 in relation to sec69 of the
Revised Administrative code. Section 37 (a) provides that aliens shall be arrested and
deported upon warrant of the Commissioner of Immigration and Deportation after a
determination by the Board of Commissioners of the existence of a ground for
deportation against them. Deportation proceedings are administrative in character and
never construed as a punishment but a preventive measure. Therefore, it need not be
conducted strictly in accordance with ordinary Court proceedings. What is essential is
that there should be a specific charge against the alien intended to be arrested and
deported. A fair hearing must also be conducted with assistance of a counsel if desired.
Lastly, the power to deport aliens is an act of the State and done under the authority of the
sovereign power. It a police measure against the undesirable aliens
whose continued presence in the country is found to be injurious to the public good and
tranquility of the people.

Alvarez vs. The Court of First Instance


64 Phil. 33 (GR No. L-45358)
January 29, 1937
J. Imperial
Facts:
On June 3, 1936, the chief of of the secret service of the Anti-Usury Board presented to
Judge David, presiding judge of CFI of Tayabas, alleging that according to reliable
information, the petitioner is keeping in his house in Infanta, Tayabas documents,
receipts, lists, chits and other papers used by him in connection with his activities as a
money lender charging usurious rates of interest in violation of the law.
In his oath the chief of the secret service did not swear to the truth of his statements upon
his knowledge of the facts but the information received by him from a reliable person.
Upon this questioned affidavit, the judge issued the search warrant, ordering the search of
the petitioners house at any time of the day or night, the seizure of the books and
documents and the immediate delivery of such to him (judge). With said warrant, several
agents of the Anti-Usury Board entered the petitioner's store and residence at 7 o'clock of
the night and seized and took possession of various articles belonging to the petitioner.
The petitioner asks that the warrant of issued by the Court of First Instance of Tayabas,
ordering the search of his house and the seizure, at anytime of the day or night, of certain
accounting books, documents, and papers belonging to him in his residence situated in
Infanta, Tayabas, as well as the order of a later date, authorizing the agents of the AntiUsury board to retain the articles seized, be declared illegal and set aside, and prays that
all the articles in question be returned to him.

Issues:
1.) What is the nature of searchers and seizures as contemplated in the law?
2.) What is required of the oath in the issuance of search warrant?
3.) What is the purpose of the disposition in addition to the affidavit?
4.) Whether or not the search warrant could be serve at night?
5.) Whether or not the seizure of evidence to use in an investigation is constitutional?
6.) Whether or not there was a waiver of constitutional guarantees?
Held:
A search warrant is an order in writing, issued in the name of the People of the Philippine
Islands, signed by a judge or a justice of the peace, and directed to a peace officer,
commanding him to search for personal property and bring it before the court (section 95,
General Orders. No. 58, as amended by section 6 of Act No. 2886). Of all the rights of a
citizen, few are of greater importance or more essential to his peace and happiness than
the right of personal security, and that involves the exemption of his private affairs,
books, and papers from the inspection and scrutiny of others (In re Pacific Railways
Commission, 32 Fed., 241; Interstate Commerce Commission vs Brimson, 38 Law. ed.,
1047; Broydvs. U. S., 29 Law. ed., 746; Caroll vs. U. S., 69 Law. ed., 543, 549). While
the power to search and seize is necessary to the public welfare, still it must be exercised
and the law enforced without transgressing the constitutional rights or citizen, for the
enforcement of no statue is of sufficient importance to justify indifference to the basis
principles of government (People vs. Elias, 147 N. E., 472).
As the protection of the citizen and the maintenance of his constitutional right is one of
the highest duties and privileges of the court, these constitutional guaranties should be
given a liberal construction or a strict construction in favor of the individual, to prevent

stealthy encroachment upon, or gradual depreciation on, the rights secured by


them(State vs.Custer County, 198 Pac., 362; State vs. McDaniel, 231 Pac., 965; 237 Pac.,
373). Since the proceeding is a drastic one, it is the general rule that statutes authorizing
searches and seizure or search warrants must be strictly construed (Rose vs. St. Clair, 28
Fed., [2d], 189; Leonard vs. U. S., 6 Fed. [2d], 353; Perry vs. U. S. 14 Fed. [2d],88;
Cofer vs. State, 118 So., 613).
Unreasonable searches and seizures are a menace against which the constitutional
guarantee afford full protection. The term "unreasonable search and seizure" is not
defined in the Constitution or in General Orders No. 58, and it is said to have no fixed,
absolute or unchangeable meaning, although the term has been defined in general
language. All illegal searches and seizure are unreasonable while lawful ones are
reasonable. What constitutes a reasonable or unreasonable search or seizure in any
particular case is purely a judicial question, determinable from a consideration of the
circumstances involved, including the purpose of the search, the presence or absence or
probable cause, the manner in which the search and seizure was made, the place or thing
searched, and the character of the articles procured (Go-Bart Importing Co. vs. U. S. 75
Law. ed., 374; Peru vs. U. S., 4 Fed., [2d], 881;U. S. vs.Vatune, 292 Fed., 497;
Angelo vs. U. S. 70 Law, ed., 145; Lambert vs. U. S. 282 Fed., 413; U. S. vs. Bateman,
278 Fed., 231; Mason vs. Rollins, 16 Fed. Cas. [No. 9252], 2 Biss., 99).
Neither the Constitution nor General Orders. No. 58 provides that it is of imperative
necessity to take the deposition of the witnesses to be presented by the applicant or
complainant in addition to the affidavit of the latter. The purpose of both in requiring the
presentation of depositions is nothing more than to satisfy the committing magistrate of
the existence of probable cause. Therefore, if the affidavit of the applicant or complainant
is sufficient, the judge may dispense with that of other witnesses. Inasmuch as the
affidavit of the agent in this case was insufficient because his knowledge of the facts was
not personal but merely hearsay, it is the duty of the judge to require the affidavit of one
or more witnesses for the purpose of determining the existence of probable cause to
warrant the issuance of the search warrant. When the affidavit of the applicant of the
complaint contains sufficient facts within his personal and direct knowledge, it is
sufficient if the judge is satisfied that there exist probable cause; when the applicant's
knowledge of the facts is mere hearsay, the affidavit of one or more witnesses having a

personal knowledge of the fact is necessary. We conclude, therefore, that the warrant
issued is likewise illegal because it was based only on the affidavit of the agent who had
no personal knowledge of the facts.
Section 101 of General Orders, No. 58 authorizes that the search be made at night when it
is positively asserted in the affidavits that the property is on the person or in the place
ordered to be searched. As we have declared the affidavits insufficient and the warrant
issued exclusively upon it illegal, our conclusion is that the contention is equally well
founded and that the search could not legally be made at night.
The only description of the articles given in the affidavit presented to the judge was as
follows: "that there are being kept in said premises books, documents, receipts, lists, chits
and other papers used by him in connection with his activities as money-lender, charging
a usurious rate of interest, in violation of the law." Taking into consideration the nature of
the article so described, it is clear that no other more adequate and detailed description
could have been given, particularly because it is difficult to give a particular description
of the contents thereof. The description so made substantially complies with the legal
provisions because the officer of the law who executed the warrant was thereby placed in
a position enabling him to identify the articles, which he did.
At the hearing of the incidents of the case raised before the court it clearly appeared that
the books and documents had really been seized to enable the Anti-Usury Board to
conduct an investigation and later use all or some of the articles in question as evidence
against the petitioner in the criminal cases that may be filed against him. The seizure of
books and documents by means of a search warrant, for the purpose of using them as
evidence in a criminal case against the person in whose possession they were found, is
unconstitutional because it makes the warrant unreasonable, and it is equivalent to a
violation of the constitutional provision prohibiting the compulsion of an accused to
testify against himself (Uy Kheytin vs. Villareal, 42 Phil,, 886; Brady vs.U. S., 266 U. S.,
620; Temperani vs. U. S., 299 Fed., 365; U. S. vs.Madden, 297 Fed., 679; Boyd vs. U.
S.,116 U. S., 116; Caroll vs. U. S., 267 U. S., 132). Therefore, it appearing that at least
nineteen of the documents in question were seized for the purpose of using them as
evidence against the petitioner in the criminal proceeding or proceedings for violation

against him, we hold that the search warrant issued is illegal and that the documents
should be returned to him.
The Anti-Usury Board insinuates in its answer that the petitioner cannot now question the
validity of the search warrant or the proceedings had subsequent to the issuance thereof,
because he has waived his constitutional rights in proposing a compromise whereby he
agreed to pay a fine of P200 for the purpose of evading the criminal proceeding or
proceedings. We are of the opinion that there was no such waiver, first, because the
petitioner has emphatically denied the offer of compromise and, second, because if there
was a compromise it reffered but to the institution of criminal proceedings fro violation
of the Anti-Usury Law. The waiver would have been a good defense for the respondents
had the petitioner voluntarily consented to the search and seizure of the articles in
question, but such was not the case because the petitioner protested from the beginning
and stated his protest in writing in the insufficient inventory furnished him by the agents.

SORIANO MATA vs HON. JOSEPHINE K. BAYONA


FACTS:
The contention is that the search warrant issued by respondent Judge was based
merely on application for Search Warrant and a joint affidavit of private respondents
which were wrongfully allegedly subscribed, and sworn to before the Clerk of Court.
Furthermore, there was allegedly a failure on the part of the respondent Judge to
attach the necessary papers pertinently under PD 810, as amended by PD1306, the
information against him alleging that Soriano Mata offered, took, and arrangedvbets
on the Jai Alai game by selling illegal tickets knows as Masiao tickets without any
authority from the Philippine Jai Alai and Corporation or from the government
authorities concerned.
ISSUE:
Whether or not Search Warrant is valid?
HELD:
No. The Search Warrant is declared as illegal.
RATIONALE:
Deposition, sometimes used in a broad sense to describe any written statement
verified by oath; but in its more technical and appropriate sense the meaning of the
word is limited to written testimony of a witness given in the course of a judicial
proceeding in advance of the trial or hearing upon oral examination.
Mere affidavits of the complainant and his witnesses are thus not sufficient. The

examining Judge has to take depositions in writing of the complainant and the
witnesses he may produce and to attach them to the record. Such written deposition
is necessary in order that the Judge may be able to property determine the existence
or non-existence of the probable cause, to hold liable for perjury the person giving if
it will be found later his declarations are false.

PEOPLE VS. DEL ROSARIO [234 SCRA 246; G.R. NO. 109633; 20 JUL
1994]
Wednesday, February 04, 2009 Posted by Coffeeholic Writes
Labels: Case Digests, Political Law

Facts: Accused was charged and convicted by the trial court of illegal possession of
firearms and illegal possession and sale of drugs, particularly methamphetamine or shabu.
After the issuance of the search warrant, which authorized the search and seizure of an
undetermined quantity of methamphetamine and its paraphernalias, an entrapment was
planned that led to the arrest of del Rosario and to the seizure of the shabu, its
paraphernalias and of a .22 caliber pistol with 3 live ammunition.

Issue: Whether or Not the seizure of the firearms was proper.

Held: No. Sec 2 art. III of the constitution specifically provides that a search warrant
must particularly describe the things to be seized. In herein case, the only objects to be
seized that the warrant determined was the methamphetamine and the paraphernalias
therein. The seizure of the firearms was unconstitutional.
Wherefore the decision is reversed and the accused is acquitted.

Umil vs. Ramos


FACTS: This consolidated case of 8 petitions for habeas corpus assails the validity of the
arrests and searches made by the military on the petitioners. The arrests relied on the
confidential information that the authorities received. Except for one case where
inciting to sedition was charged, the rest are charged with subversion for being a member
of the New Peoples Army.
RULING: The arrests were legal. Regarding the subversion cases, the arrests were legal
since subversion is a form of a continuing crime together with rebellion, conspiracy or
proposal to commit rebellion/subversion, and crimes committed in furtherance thereof or
in connection therewith. On the inciting to sedition case, the arrest was legal since an
information was filed prior to his arrest. Lastly, the arrests were not fishing expeditions
but a result of an in-depth surveillance of NPA safe houses pinpointed by none other than
members of the NPA.
The right to preliminary investigation should be exercised by the offender as soon as
possible. Otherwise, it would be considered as impliedly waived and the filing of
information can proceed. This sort of irregularity is not sufficient to set aside a valid
judgment upon a sufficient complaint and after a trial free from error.
DISSENT: (Sarmiento, J.) The confidential information was nothing but hearsay. The
searches and arrests made were bereft of probable cause and that the petitioners were not
caught in flagrante delicto or in any overt act. Utmost, the authorities was lucky in their
fishing expeditions.
2. The Bill of Rights can only be invoked only against the state. People vs. Marti
--Marti and his wife went to the booth of the "Manila Packing and Export Forwarders"
carrying with them four (4) gift-wrapped packages. Marti informed the owner that the
packages simply contained books, cigars and gloves as gifts to his friends in Zurich and
refused to allow the owner to examine and inspect the packages. However, before the
delivery of the box to the Bureau of Customs, the owner's husband inspected the package
and found marijuana which was later turned over to the NBI. A case was filed against
Marti. Marti invoked his right against illegal searches and seizure. Held: The
constitutional proscription against unlawful searches and seizures therefore applies as a
restraint directed only against the government and its agencies tasked with the
enforcement of the law. Thus, it could only be invoked against the State to whom the
restraint against arbitrary and unreasonable exercise of power is imposed.
Corollarily, alleged violations against unreasonable search and seizure may only be
invoked against the State by an individual unjustly traduced by the exercise of sovereign
authority. To agree with appellant that an act of a private individual in violation of the
Bill of Rights should also be construed as an act of the State would result in serious legal
complications and an absurd interpretation of the constitution

PEOPLE VS. SUCRO [195 SCRA 388; G.R. No. 93239; 18 Mar 1991]
Wednesday, February 04, 2009 Posted by Coffeeholic Writes
Labels: Case Digests, Political Law
Facts: Pat. Fulgencio went to Arlie Regalados house at C. Quimpo to monitor activities
of Edison SUCRO (accused). Sucro was reported to be selling marijuana at a chapel 2
meters away from Regalados house. Sucro was monitored to have talked and exchanged
things three times. These activities are reported through radio to P/Lt. Seraspi. A third
buyer was transacting with appellant and was reported and later identified as Ronnie
Macabante. From that moment, P/Lt.Seraspi proceeded to the area. While the police
officers were at the Youth Hostel in Maagama St. Fulgencio told Lt. Seraspi to intercept.
Macabante was intercepted at Mabini and Maagama crossing in front of Aklan Medical
center. Macabante saw the police and threw a tea bag of marijuana on the ground.
Macabante admitted buying the marijuana from Sucro in front of the chapel.
The police team intercepted and arrested SUCRO at the corner of C. Quimpo and
Veterans. Recovered were 19 sticks and 4 teabags of marijuana from a cart inside the
chapel and another teabag from Macabante.

Issues:
(1) Whether or Not arrest without warrant is lawful.
(2) Whether or Not evidence from such arrest is admissible.

Held: Search and seizures supported by a valid warrant of arrest is not an absolute rule.
Rule 126, Sec 12 of Rules of Criminal Procedure provides that a person lawfully arrested
may be searched for dangerous weapons or anything, which may be used as proff of the
commission of an offense, without a search warrant.(People v. Castiller) The failure of
the police officers to secure a warrant stems from the fact that their knowledge required
from the surveillance was insufficient to fulfill requirements for its issuance. However,
warantless search and seizures are legal as long as PROBABLE CAUSE existed. The
police officers have personal knowledge of the actual commission of the crime from the
surveillance of the activities of the accused. As police officers were the ones conducting
the surveillance, it is presumed that they are regularly in performance of their duties.

Go Vs Court of Appeals G.R. No. 101837 Feb. 11, 1992


FACTS:On July 2 1991, Eldon Maguan was allegedly shot to death by accused Rolito Go
due to a traffic altercation whenpetitioners car and the victims car nearly bumped
each other. The Security Guard of the Cravings Bake Shop saw the wholeincident
and point herein petitioner as the gunman, which he positively identified when questioned
by the authorities. Beingconvinced of the suspects identity, the police launched a
manhunt operation that caused petitioner to present himself before theSan Juan Police
Station to verify the said issue; he was then detained by the police.
ISSUE:Whether or not herein petitioners arrest valid?
RULING:The reliance of both petitioner and the Solicitor General upon Umil v.Ramos is,
in the circumstances of this case,misplaced. In the instant case, the offense for which
petitioner was arrested was murder, an offense which was obviouslycommenced and
completed at one definite location in time and space. No one had pretended that the fatal
shooting of Maguan wasa "continuing crime."..... none of the "arresting" officers had any
"personal knowledge" of facts indicating that petitioner was the gunmanwho had shot
Maguan. The information upon which the police acted had been derived from statements
made by allegedeyewitnesses to the shooting one stated that petitioner was the
gunman; another was able to take down the alleged gunman'scar's plate number which
turned out to be registered in petitioner's wife's name. That information did not, however,
constitute"personal knowledge."

GO VS. COURT OF APPEALS [206 SCRA 138; G.R. NO. 101837; 11 FEB
1992]
Facts: Petitioner, while traveling in the wrong direction on a one-way street, almost had a
collision with another vehicle. Petitioner thereafter got out of his car, shot the driver of
the other vehicle, and drove off. An eyewitness of the incident was able to take down
petitioners plate number and reported the same to the police, who subsequently ordered a
manhunt for petitioner. 6 days after the shooting, petitioner presented himself in the
police station, accompanied by 2 lawyers, the police detained him. Subsequently a
criminal charge was brought against him. Petitioner posted bail, the prosecutor filed the

case to the lower court, setting and commencing trial without preliminary investigation.
Prosecutor reasons that the petitioner has waived his right to preliminary investigation as
bail has been posted and that such situation, that petitioner has been arrested without a
warrant lawfully, falls under Section 5, Rule 113 and Section 7, Rule 112 of The 1985
Rules of Criminal Procedure which provides for the rules and procedure pertaining to
situations of lawful warrantless arrests. Petitioner in his petition for certiorari assails such
procedure and actions undertaken and files for a preliminary investigation.
Issues:
(1) WON warrantless arrest of petitioner was lawful.
(2) WON petitioner effectively waived his right to preliminary investigation.
Held: Petitioner and prosecutor err in relying on Umil v. Ramos, wherein the Court
upheld the warrantless arrest as valid effected 1 to 14 days from actual commission of the
offenses, which however constituted continuing crimes, i.e. subversion, membership in
an outlawed organization, etc. There was no lawful warrantless arrest under Section 5,
Rule 113. This is because the arresting officers were not actually there during the
incident, thus they had no personal knowledge and their information regarding petitioner
were derived from other sources. Further, Section 7, Rule 112, does not apply.
Petitioner was not arrested at all, as when he walked in the police station, he neither
expressed surrender nor any statement that he was or was not guilty of any crime. When a
complaint was filed to the prosecutor, preliminary investigation should have been
scheduled to determine probable cause. Prosecutor made a substantive error, petitioner is
entitled to preliminary investigation, necessarily in a criminal charge, where the same is
required appear thereat. Petition granted, prosecutor is ordered to conduct preliminary
investigation, trial for the criminal case is suspended pending result from preliminary
investigation, petitioner is ordered released upon posting a bail bond.

People v. Rodriguez Case Brief


Criminal Law & Criminal
Procedure Add Comment
Summary of People v.
Rodriguez, District Ct. App.
CA 1960
Facts: mrs. Rodriguez left her
four children unattended in their
home in LA. A fire broke out in the home late in the evening. Three children were pulled
out of the fire by neighbors, but Carlos, age two, was lost. Defendant was found near a
bar, although she had not been drinking, she stated knowledge of the fire.

Issue: Whether the evidence is sufficient to support a finding of involuntary


manslaughter?
(Was there evidence of criminal intent or criminal negligence?)
Holding: No
Procedure: Bench trial found def guilty of involuntary manslaughter. New trial denied.
Order and judgment reversed.
Rule: Involuntary manslaughter is the unlawful killing of a human being without malice,
in the commission of a crime, not a felony, or in the commission of a lawful act which
might produce death, in an unlawful manner, or w/o due caution and circumspect . . .
Criminal negligence is a required element of this crime. Criminal negligence is
knowledge, actual or imputed, that the act of the slayer tended to endanger life and that
the fatal consequences of the negligent act could reasonably have been foreseeable.
Court Rationale: There was no evidence that the death was the result of natural and
probable result of a criminally negligent act. There was no evidence def realized her
conduct would in all probability produce death. There was no evidence connecting def in
any way to the fire. Mere negligence is not sufficient to authorize a conviction of invol.
manslaughter.

G.R. No. 89139 August 2, 1990ROMEO POSADAS y ZAMORA,


petitioner,vs.
THE HONORABLE COURT OF APPEALS and THE PEOPLE OF THE PHILIPPINES,
respondents.
GANCAYCO,
J.:
Facts:While Pat. Ungab and Umpar were conducting a surveillance along Magallanes
Street,Davao City, they spotted petitioner carrying a "buri" bag and they noticed him to
beacting suspiciously.They approached the petitioner and identified themselves
asmembers of the INP. Petitioner attempted to flee but his attempt to get away
wasthwarted by the two notwithstanding his resistance.They then checked the "buri" bag
of the petitioner where they found one (1) caliber .38revolver, two (2) rounds of live
ammunition for a .38 caliber gun 2 a smoke (tear gas)grenade, 3 and two (2) live
ammunitions for a .22 caliber gun. 4 the petitioner was askedto show the necessary
license or authority to possess the firearms and ammunitions butfailed to do so.
Issue:Whether or not the warantless arrest and search was valid.

Ruling:An arrest without a warrant may be effected by a peace officer or private person,
amongothers, when in his presence the person to be arrested has committed, is
actuallycommitting, or is attempting to commit an offense; or when an offense has in fact
justbeen committed, and he has personal knowledge of the facts indicating that the
personarrested has committed it.Contrary to the argument of the Solicitor General that
when the two policemenapproached the petitioner, he was actually committing or had just
committed the offenseof illegal possession of firearms and ammunitions in the presence
of the police officersand consequently the search and seizure of the contraband was
incidental to the lawfularrest in accordance with Section 12, Rule 126 of the 1985 Rules
on CriminalProcedure; At the time the peace officers in this case identified themselves
andapprehended the petitioner as he attempted to flee they did not know that he
hadcommitted, or was actually committing the offense of illegal possession of firearms
andammunitions. They just suspected that he was hiding something in the buri bag.
Theydid not know what its contents were. The said circumstances did not justify an
arrestwithout a warrant.

ARUTA
In the morning of 13 Dec 1988, the law enforcement officers received information from
an informant named Benjie that a certain Aling Rosa would be leaving forBaguio
City on 14 Dec 1988 and would be back in the afternoon of the same day carrying with
her a large volume of marijuana; At 6:30 in the evening of 14 Dec 1988, Aruta alighted
from a Victory Liner Bus carrying a travelling bag even as the informant pointed her out
to the law enforcement officers; NARCOM officers approached her and introduced
themselves as NARCOM agents; When asked by Lt. Abello about the contents of her

travelling bag, she gave the same to him; When they opened the same, they found dried
marijuana leaves; Aruta was then brought to the NARCOM office for investigation.
ISSUE: Whether or not the conducted search and seizure is constitutional.
HELD: The SC ruled in favor of Aruta and has noted that some drug traffickers are being
freed due to technicalities. Aruta cannot be said to be committing a crime. Neither was
she about to commit one nor had she just committed a crime. Aruta was merely crossing
the street and was not acting in any manner that would engender a reasonable ground for
the NARCOM agents to suspect and conclude that she was committing a crime. It was
only when the informant pointed to Aruta and identified her to the agents as the carrier of
the marijuana that she was singled out as the suspect. The NARCOM agents would not
have apprehended Aruta were it not for the furtive finger of the informant because, as
clearly illustrated by the evidence on record, there was no reason whatsoever for them to
suspect that accused-appellant was committing a crime, except for the pointing finger of
the informant. The SC could neither sanction nor tolerate as it is a clear violation of the
constitutional guarantee against unreasonable search and seizure. Neither was there any
semblance of any compliance with the rigid requirements of probable cause and
warrantless arrests. Consequently, there was no legal basis for the NARCOM agents to
effect a warrantless search of Arutas bag, there being no probable cause and the accusedappellant not having been lawfully arrested. Stated otherwise, the arrest being incipiently
illegal, it logically follows that the subsequent search was similarly illegal, it being not
incidental to a lawful arrest. The constitutional guarantee against unreasonable search and
seizure must perforce operate in favor of accused-appellant. As such, the articles seized
could not be used as evidence against accused-appellant for these are fruits of a
poisoned tree and, therefore, must be rejected, pursuant to Article III, Sec. 3(2) of the
Constitution.

PEOPLE VS. AMMINUDIN [163 SCRA 402; G.R. L-74869; 6 Jul 1988]
Friday, February 06, 2009 Posted by Coffeeholic Writes
Labels: Case Digests, Political Law
Facts: Idel Aminnudin, accused-appellant was arrested on June 25, 1984, shortly after
disembarking from the M/V Wilcon 9 at about 8:30 in the evening, in Iloilo City. The PC
officers who were in fact waiting for him because of a tip from one their informers
simply accosted him, inspected his bag and finding what looked liked marijuana leaves
took him to their headquarters for investigation. The two bundles of suspect articles were

confiscated from him and later taken to the NBI laboratory for examination. It was found
to contain three kilos of what were later analyzed as marijuana leaves by an NBI
forensic examiner. An information for violation of theDangerous Drugs Act was filed
against him. Later, the information was amended to include Farida Ali y Hassen, who had
also been arrested with him that same evening and likewise investigated. Both were
arraigned and pleaded not guilty. Subsequently, the fiscal filed a motion to dismiss the
charge against Ali on the basis of a sworn statement of the arresting officers absolving her
after a 'thorough investigation." The motion was granted, and trial proceeded only against
the accused-appellant, who was eventually convicted . In his defense, Aminnudin
disclaimed the marijuana, averring that all he had in his bag was his clothing consisting
of a jacket, two shirts and two pairs of pants. He alleged that he was
arbitrarily arrested and immediately handcuffed. His bag was confiscated without a
search warrant. At the PC headquarters, he was manhandled to force him to admit he was
carrying the marijuana, the investigator hitting him with a piece of wood in the chest and
arms even as he parried the blows while he was still handcuffed. He insisted he did not
even know what marijuana looked like and that his business was selling watches and
sometimes cigarettes. However the RTC rejected his allegations. Saying that he only has
two watches during that time and that he did not sufficiently proved the injuries allegedly
sustained.

Issue: Whether or not search of defendants bag is legal.

Held: The search was illegal. Defendant was not caught in flagrante delicto, which could
allow warrantless arrest or search. At the moment of his arrest, he was not committing a
crime. Nor was he about to do so or had just done so. To all appearances, he was like any
of the other passengers innocently disembarking from the vessel. The said marijuana
therefore could not be appreciated as evidence against the defendant, and furthermore he
is acquitted of the crime as charged.

PEOPLE VS. MALMSTEDT [198 SCRA 401; G.R. No. 91107; 19 Jun 1991]
Friday, February 06, 2009 Posted by Coffeeholic Writes
Labels: Case Digests, Political Law
Facts: In an information filed against the accused- appellant Mikael Malmstead was

charged before the RTC of La Trinidad, Benguet, for violation of Section 4, Art. II of
Republic Act 6425, as amended, otherwise known as the Dangerous Drugs Act of 1972,
as amended.
Accused Mikael Malmstedt, a Swedish national, entered the Philippines for the third time
in December 1988 as a tourist. He had visited the country sometime in 1982 and 1985.
In the evening of 7 May 1989, accused left for Baguio City. Upon his arrival thereat in
the morning of the following day, he took a bus to Sagada and stayed in that place for two
(2) days. Then in the 7 in the morning of May 11, 1989, the accused went to Nangonogan
bus stop in Sagada.
At about 8: 00 o'clock in the morning of that same day (11 May 1989), Captain Alen
Vasco, the Commanding Officer of the First Regional Command (NARCOM) stationed at
Camp Dangwa, ordered his men to set up a temporary checkpoint at Kilometer 14, Acop,
Tublay, Mountain Province, for the purpose of checking all vehicles coming from the
Cordillera Region. The order to establish a checkpoint in the said area was prompted by
persistent reports that vehicles coming from Sagada were transporting marijuana and
other prohibited drugs. Moreover, information was received by the Commanding Officer
of NARCOM, that same morning that a Caucasian coming from Sagada had in his
possession prohibited drugs. The group composed of seven (7) NARCOM officers, in
coordination with Tublay Police Station, set up a checkpoint at the designated area at
about 10:00 o'clock in the morning and inspected all vehicles coming from the Cordillera
Region.
The two (2) NARCOM officers started their inspection from the front going towards the
rear of the bus. Accused who was the sole foreigner riding the bus was seated at the rear
thereof.
During the inspection, CIC Galutan noticed a bulge on accused's waist. Suspecting the
bulge on accused's waist to be a gun, the officer asked for accused's passport and other
identification papers. When accused failed to comply, the officer required him to bring
out whatever it was that was bulging on his waist. The bulging object turned out to be a
pouch bag and when accused opened the same bag, as ordered, the officer noticed four
(4) suspicious-looking objects wrapped in brown packing tape, prompting the officer to
open one of the wrapped objects. The wrapped objects turned out to contain hashish, a
derivative of marijuana.
Thereafter, accused was invited outside the bus for questioning. But before he alighted

from the bus, accused stopped to get two (2) travelling bags from the luggage carrier.
Upon stepping out of the bus, the officers got the bags and opened them. A teddy bear
was found in each bag. Feeling the teddy bears, the officer noticed that there were bulges
inside the same which did not feel like foam stuffing. It was only after the officers had
opened the bags that accused finally presented his passport.
Accused was then brought to the headquarters of the NARCOM at Camp Dangwa, La
Trinidad, Benguet for further investigation. At the investigation room, the officers opened
the teddy bears and they were found to also contain hashish. Representative samples were
taken from the hashish found among the personal effects of accused and the same were
brought to the PC Crime Laboratory for chemical analysis.
In the chemistry report, it was established that the objects examined were hashish. a
prohibited drug which is a derivative of marijuana. Thus, an information was filed against
accused for violation of the Dangerous Drugs Act.
ACCUSEDS DEFENSE
During the arraignment, accused entered a plea of "not guilty." For his defense, he raised
the issue of illegal search of his personal effects. He also claimed that the hashish was
planted by the NARCOM officers in his pouch bag and that the two (2) travelling bags
were not owned by him, but were merely entrusted to him by an Australian couple whom
he met in Sagada. He further claimed that the Australian couple intended to take the same
bus with him but because there were no more seats available in said bus, they decided to
take the next ride and asked accused to take charge of the bags, and that they would meet
each other at the Dangwa Station.
The trial court found the guilt of the accused Mikael Malmstedt established beyond
reasonable doubt.
Seeking the reversal of the decision of the trial court finding him guilty of the crime
charged, accused argues that the search of his personal effects was illegal because it was
made without a search warrant and, therefore, the prohibited drugs which were
discovered during the illegal search are not admissible as evidence against him.

Issue: Whether or Not the contention of the accused is valid, and therefore the RTC
ruling be reversed.

Held: The Constitution guarantees the right of the people to be secure in their persons,
houses, papers and effects against unreasonable searches and seizures. However, where
the search is made pursuant to a lawful arrest, there is no need to obtain a search warrant.
A lawful arrest without a warrant may be made by a peace officer or a private person
under the following circumstances.
Sec. 5 Arrest without warrant; when lawful. A peace officer or a private person may,
without a warrant, arrest a person:
(a) When, in his presence, the person to be arrested has committed is actually committing,
or is attempting to commit an offense;
(b) When an offense has in fact just been committed, and he has personal knowledge of
facts indicating that the person to be arrested has committed it; and
(c) When the person to be arrested is a prisoner who has escaped from a penal
establishment or place where he is serving final judgment or temporarily confined while
his case is pending, or has escaped while being transferred from one confinement to
another.
Accused was searched and arrested while transporting prohibited drugs (hashish). A
crime was actually being committed by the accused and he was caught in flagrante
delicto. Thus, the search made upon his personal effects falls squarely under paragraph
(1) of the foregoing provisions of law, which allow a warrantless search incident to a
lawful arrest. While it is true that the NARCOM officers were not armed with a search
warrant when the search was made over the personal effects of accused, however, under
the circumstances of the case, there was sufficient probable cause for said officers to
believe that accused was then and there committing a crime.
Probable cause has been defined as such facts and circumstances which could lead a
reasonable, discreet and prudent man to believe that an offense has been committed, and
that the objects sought in connection with the offense are in the place sought to be
searched. Warrantless search of the personal effects of an accused has been declared by
this Court as valid, because of existence of probable cause, where the smell of marijuana
emanated from a plastic bag owned by the accused, 10 or where the accused was acting
suspiciously, 11 and attempted to flee.

The appealed judgment of conviction by the trial court is hereby affirmed. Costs against
the accused-appellant.

PEOPLE VS. MUSA [217 SCRA 597; G.,R. NO. 96177; 27 JAN 1993]
Friday, February 06, 2009 Posted by Coffeeholic Writes
Labels: Case Digests, Political Law
Facts: A civilian informer gave the information that Mari Musa was engaged in selling
marijuana in Suterville, Zamboanga City. Sgt. Ani was ordered by NARCOM leader
T/Sgt. Belarga, to conduct a surveillance and test buy on Musa. The civilian informer
guided Ani to Musas house and gave the description of Musa. Ani was able to buy
one newspaper-wrapped dried marijuana for P10.00.
The next day, a buy-bust was planned. Ani was to raise his right hand if
hesuccessfully buys marijuana from Musa. As Ani proceeded to the house, the NARCOM
team positioned themselves about 90 to 100 meters away. From his position, Belarga
could see what was going on. Musa came out of the house and asked Ani what he wanted.
Ani said he wanted more marijuana and gave Musa the P20.00 marked money. Musa
went into the house and came back, giving Ani two newspaper wrappers containing dried
marijuana. Ani opened and inspected it. He raised his right hand as a signal to the other
NARCOM agents, and the latter moved in and arrested Musa insidethe house. Belarga
frisked Musa in the living room but did not find the marked money (gave it to his wife
who slipped away). T/Sgt. Belarga and Sgt. Lego went to the kitchen and found a
cellophane colored white and stripe hanging at the corner of the kitchen. They asked
Musa about its contents but failed to get a response. So they opened it and found dried
marijuana leaves inside. Musa was then placed under arrest.

Issue: Whether or Not the seizure of the plastic bag and the marijuana inside it is
unreasonable, hence, inadmissible as evidence.

Held: Yes. It constituted unreasonable search and seizure thus it may not be admitted as
evidence. The warrantless search and seizure, as an incident to a suspects lawful arrest,
may extend beyond the person of the one arrested to include the premises or surroundings

under his immediate control. Objects in the plain view of an officer who has the right to
be in the position to have that view are subject to seizure and may be presented as
evidence. The plain view doctrine is usually applied where a police officer is not
searching for evidence against the accused, but nonetheless inadvertently comes across an
incriminating object. It will not justify the seizure of the object where the incriminating
nature of the object is not apparent from the plain view of the object.
In the case at bar, the plastic bag was not in the plain view of the police. They arrested
the accused in the living room and moved into the kitchen in search for other evidences
where they found the plastic bag. Furthermore, the marijuana inside the plastic bag was
not immediately apparent from the plain view of said object.
Therefore, the plain view does not apply. The plastic bag was seized illegally and cannot
be presented in evidence pursuant to Article III Section 3 (2) of the Constitution.

Valmonte vs. De VillaFacts:


On 20 January 1987, the National Capital Region District Command (NCRDC)
wasactivated pursuant to Letter of Instruction 02/87 of the Philippine General
Headquarters,AFP, with the mission of conducting security operations within its area of
responsibility andperipheral areas, for the purpose of establishing an effective territorial
defense, maintainingpeace and order, and providing an atmosphere conducive to the
social, economic andpolitical development of the National Capital Region. As part of its
duty to maintain peaceand order, the NCRDC installed checkpoints in various parts of
Valenzuela, Metro Manila.Petitioners aver that, because of the installation of said
checkpoints, the residents of Valenzuela are worried of being harassed and of their safety
being placed at the arbitrary,capricious and whimsical disposition of the military manning
the checkpoints, consideringthat their cars and vehicles are being subjected to regular
searches and check-ups, especiallyat night or at dawn, without the benefit of a search
warrant and/or court order. Their allegedfear for their safety increased when, at dawn of 9

July 1988, Benjamin Parpon, a supplyofficer of the Municipality of Valenzuela, Bulacan,


was gunned down allegedly in cold bloodby the members of the NCRDC manning the
checkpoint along McArthur Highway at Malinta,Valenzuela, for ignoring and/or refusing
to submit himself to the checkpoint and forcontinuing to speed off inspire of warning
shots fired in the air.
Issue:
WON the installation of checkpoints violates the right of the people againstunreasonable
searches and seizures
Held:
Petitioner's concern for their safety and apprehension at being harassed by themilitary
manning the checkpoints are not sufficient grounds to declare the checkpoints per se
, illegal. No proof has been presented before the Court to show that, in the course of
theirroutine checks, the military, indeed, committed specific violations of petitioners''
rightsagainst unlawful search and seizure of other rights. The constitutional right
againstunreasonable searches and seizures is a personal right invocable only by those
whose rightshave been infringed, or threatened to be infringed. Not all searches and
seizures areprohibited. Those which are reasonable are not forbidden. The setting up of
the questionedcheckpoints may be considered as a security measure to enable the
NCRDC to pursue itsmission of establishing effective territorial defense and maintaining
peace and order for thebenefit of the public. Checkpoints may not also be regarded as
measures to thwart plots todestabilize the govt, in the interest of public security. Between
the inherent right of thestate to protect its existence and promote public welfare and an
individuals right against awarrantless search w/c is, however, reasonably conducted, the
former should prevail. True,the manning of checkpoints by the military is susceptible of
abuse by the military in thesame manner that all governmental power is susceptible of
abuse. But, at the cost of occasional inconvenience, discomfort and even irritation to the
citizen, the checkpointsduring these abnormal times, when conducted w/in reasonable
limits, are part of the pricewe pay for an orderly society and a peaceful community.

In 1996, Dominador Samiano, Jr., an agent of the National Bureau of Investigation (NBI)
conducted a surveillance against Maxicorp, Inc. He observed that Microsoft Softwares
(Windows Operating Systems) were being produced and packaged within the premises of
Maxicorp. Samiano, together with a civilian witness (John Benedict Sacriz) then bought a
computer unit from Maxicorp. The unit was pre-installed with a pirated copy of
Windows. For their purchase, they were issued a receipt, however, the receipt was in the
name of a certain Joel Diaz. Subsequently, Samiano applied for a search warrant before
the RTC. He brought with him Sacriz as witness. He also brought the computer unit they
bought as evidence as well as the receipt. He even added an additional witness

(Felixberto Pante), a computer technician, who showed the judge that the software in the
computer unit bought by Samiano from Maxicorp was pirated. The RTC judge,
convinced that there is a probable cause for a case of copyright infringement and unfair
competition committed by Maxicorp, issued the corresponding warrant. Maxicorp
assailed the legality of the warrant before the Court of Appeals. The Court of Appeals
ruled in favor of Maxicorp and in its decision it highlighted the fact that the receipt issued
was not in Samianos or Sacriz name hence the proceeding in the trial court was infirm
from the onset.
ISSUE: Whether or not the Court of Appeals is correct.
HELD: No. The testimonies of the two witnesses, coupled with the object and
documentary evidence they presented, are sufficient to establish the existence
ofprobable cause. From what they have witnessed, there is reason to believe that
Maxicorp engaged in copyright infringement and unfair competition to the prejudice of
Microsoft. Both NBI Agent Samiano and Sacriz were clear and insistent that the
counterfeit software were not only displayed and sold within Maxicorps premises, they
were also produced, packaged and in some cases, installed there.
The fact that the receipt issued was not in Samianos name nor was it in Sacriz name
does not render the issuance of the warrant void. No law or rule states that probablecause
requires a specific kind of evidence. No formula or fixed rule for its determination
exists. Probable cause is determined in the light of conditions obtaining in a given
situation.Thus, it was improper for the Court of Appeals to reverse the RTCs findings
simply because the sales receipt evidencing NBI Agent Samianos purchase of counterfeit
goods is not in his name.

People v Lagman, et. Al, 66 Phil. 13Facts:


In these two cases (G.R. Nos. 45892 and 45893), the appellants Tranquilino Lagman and
Primitivode Sosa are charged with a violation of section 60 of Commonwealth Act No. 1,
known as the NationalDefense Law. It is alleged that these two appellants, being Filipinos
and having reached the age of twenty years in 1936, willfully and unlawfully refused to
register in the military service between the 1stand 7th of April of said year,
notwithstanding the fact that they had been required to do so. Theevidence shows that
these two appellants were duly notified by the corresponding authorities to appearbefore
the Acceptance Board in order to register for military service in accordance with law, and

thatthe said appellants, in spite of these notices, had not registered up to the date of filing
of theinformation.The appellants do not deny these facts, but they allege in defense that
they have not registered in themilitary service because Primitivo de Sosa is fatherless and
has a mother and a brother eight years old tosupport, and Tranquilino Lagman also has a
father to support, has no military leanings, and does notwish to kill or be killed.Each of
these appellants was sentenced by the Court of First Instance to one month and one day
of imprisonment, with the costs.In this instance, the validity of the National Defense Law,
under which the accused were sentenced, isimpugned on the ground that it is
unconstitutional.
Held:
The Court held that The National Defense Law, in so far as it establishes compulsory
militaryservice, does not go against this constitutional provision but is, on the contrary, in
faithful compliancetherewith. The duty of the Government to defend the State cannot be
performed except through anarmy. To leave the organization of an army to the will of the
citizens would be to make this duty of theGovernment excusable should there be no
sufficient men who volunteer to enlist therein.The right of the Government to require
compulsory military service is a consequence of its duty todefend the State and is
reciprocal with its duty to defend the life, liberty, and property of the citizen. Inthe case of
Jacobson vs. Massachusetts (197 U.S., 11; 25 Sup. Ct. Rep., 385), it was said that,
withoutviolating the Constitution, a person may be compelled by force, if need be,
against his will, against hispecuniary interests, and even against his religious or political
convictions, to take his place in the ranksof the army of this country, and risk the chance
of being shot down in its defense. In the case of UnitedStates vs. Olson (253 Feb., 233), it
was also said that this is not deprivation of property without dueprocess of law, because,
in its just sense, there is no right of property to an office or employment.
Thecircumstance that these decisions refer to laws enacted by reason of the actual
existence of war doesnot make our case any different, inasmuch as, in the last analysis,
what justifies compulsory militaryservice is the defense of the State, whether actual or
whether in preparation to make it more effective,in case of need.

Pp vs. Marti [193 SCRA 57


Facts:
On August 14, 1987, the appellant and his common-law wife, Shirley Reyes went to
Manila Packaging and Export Forwarders to send packages to Zurich, Switzerland. It was
received by Anita Reyes and ask if she could inspect the packages. Shirley refused and
eventually convinced Anita to seal the package making it ready for shipment. Before
being sent out for delivery, Job Reyes, husband of Anita and proprietor of the courier
company, conducted an inspection of the package as part of standard operating
procedures. Upon opening the package, he noticed a suspicious odor which made him
took sample of the substance he found inside. He reported this to the NBI and invited
agents to his office to inspect the package. In the presence of the NBI agents, Job Reyes
opened the suspicious package and found dried-marijuana leaves inside. A case was filed
against Andre Marti in violation of R.A. 6425 and was found guilty by the court a quo.

Andre filed an appeal in the Supreme Court claiming that his constitutional right of
privacy was violated and that the evidence acquired from his package was inadmissible as
evidence against him.
Issue:
Can the Constitutional Right of Privacy be enforced against private individuals?
Ruling:
The Supreme Court held based on the speech of Commissioner Bernas that the Bill of
Rights governs the relationship between the individual and the state.
The constitutional proscription against unlawful searches and seizures therefore applies
as a restraint directed only against the government and its agencies tasked with the
enforcement of the law. It is not meant to be invoked against acts of private individuals. It
will be recalled that Mr Job Reyes was the one who opened the box in the presence of the
NBI agents in his place of business. The mere presence of the NBI agents did not convert
the reasonable search effected by Mr. Reyes into a warrantless search and siezure
proscribed by the constitution. Merely to observe and look at that which is in plain sight
is not a search.
The judgement of conviction finding appeallant guilty beyond reasonable doubt of the
crime charged was AFFIRMED.
b. Waterous Drug vs. NLRC [280 SCRA 735 ; G.R.No. 113271, 16 Oct. 1997]
G.R. No. 113271. October 16, 1997
Facts: Antonia Melodia Catolico was hired as a pharmacist by Waterous Drug Corp.
YSP Inc., a supplier of medicine, sold to Waterous, thru Catolico, 10 bottles of Voren
Tablets at P384 per unit. However, previews P.O.s issued to YSP, Inc. showed that the
price per bottle is P320.00. Verification was made to YSP, Inc. to determine the
discrepancy and it was found that the cost per bottle was indeed overpriced.
YSP, Inc. Accounting Department (Ms. Estelita Reyes) confirmed that the difference
represents refund of jack-up price of ten bottles of Voren tablets per sales invoice, which
was paid to Ms. Catolico. Said check was sent in an envelope addressed to Catolico.
Catolico denied receiving the same. However, Saldana, the clerk of Waterous Drug Corp.
confirmed that she saw an open envelope with a check amounting P640 payable to
Catolico.
Waterous Drug Corp. ordered the termination of Catolico for acts of dishonesty.

NLRC: Dismissed the Petition. Evidence of respondents (check from YSP) being
rendered inadmissible, by virtue of the constitutional right invoked by complainants.
Petitioners: In the light of the decision in the People v. Marti, the constitutional
protection against unreasonable searches and seizures refers to the immunity of ones
person from interference by government and cannot be extended to acts committed by
private individuals so as to bring it within the ambit of alleged unlawful intrusion by the
government.
Issue: W/N the check is admissible as evidence
Held: Yes.
Ratio: (People vs. Marti) Marti ruling: The Bill of Rights does not protect citizens from
unreasonable searches and seizures perpetrated by private individuals.
It is not true, as counsel for Catolico claims, that the citizens have no recourse against
such assaults. On the contrary, and as said counsel admits, such an invasion gives rise to
both criminal and civil liabilities. Despite this, the SC ruled that there was insufficient
evidence of cause for the dismissal of Catolico from employment Suspicion is not among
the valid causes provided by the Labor Code for the termination of Employment.

d. Burgos vs. Chief of Staff [133 SCRA 800; G.R. No. 64261; 26 Dec. 1984]
Facts:
Two warrants were issued against petitioners for the search on the premises of
Metropolitan Mail and We Forum newspapers and the seizure of items alleged to
have been used in subversive activities. Petitioners prayed that a writ of preliminary
mandatory and prohibitory injunction be issued for the return of the seized articles, and
that respondents be enjoined from using the articles thus seized as evidence against
petitioner.
Petitioners questioned the warrants for the lack of probable cause and that the two
warrants issued indicated only one and the same address. In addition, the items seized
subject to the warrant were real properties.
Issue:
Whether or not the two warrants were valid to justify seizure of the items.
Held:
The defect in the indication of the same address in the two warrants was held by the court
as a typographical error and immaterial in view of the correct determination of the place
sought to be searched set forth in the application. The purpose and intent to search two
distinct premises was evident in the issuance of the two warrant.

As to the issue that the items seized were real properties, the court applied the principle
in the case of Davao Sawmill Co. v. Castillo, ruling that machinery which is movable by
nature becomes immobilized when placed by the owner of the tenement, property or
plant, but not so when placed by a tenant, usufructuary, or any other person having only a
temporary right, unless such person acted as the agent of the owner. In the case at bar,
petitioners did not claim to be the owners of the land and/or building on which the
machineries were placed. This being the case, the machineries in question, while in fact
bolted to the ground remain movable property susceptible to seizure under a search
warrant.
However, the Court declared the two warrants null and void.
Probable cause for a search is defined as such facts and circumstances which would lead
a reasonably discreet and prudent man to believe that an offense has been committed and
that the objects sought in connection with the offense are in the place sought to be
searched.
The Court ruled that the affidavits submitted for the application of the warrant did not
satisfy the requirement of probable cause, the statements of the witnesses having been
mere generalizations.
Furthermore, jurisprudence tells of the prohibition on the issuance of general warrants.
(Stanford vs. State of Texas). The description and enumeration in the warrant of the items
to be searched and seized did not indicate with specification the subversive nature of the
said items.

e. Tambasen vs. Pp [246 SCRA 184; G.R. No. 89103; 14 July 1995]
Facts: In August 1988, P/Sgt. Natuel applied for issuance of search warrant alleging that
he received information that Petitioner had in hispossession at his house M-16 Armalite
rifles, hand grenades, .45 Cal. pistols, dynamite sticks and subversive documents, which
were used or intended to be used for illegal purposes. The application was granted.
In September, a police team, searched the house of petitioner and seized
2envelopes containing P14000, handset with antennae, transceiver with antennae,
regulator supply, academy notebook and assorted papers and handset battery pack. In
October, petitioner moved that the search andseizure be declared illegal and that the
seized articles be returned to him. In December, MTCC, in its order, directed Lt. Col.
Torres to return the money seized to petitioner ruling that any seizure should be limited to
the specified items covered thereby. SolGen petitioned with the RTC for the annulment of
the order of MTCC citing that pending the determination of legality of seizure of the
articles, they should remain in custogia legis. RTC granted the petition.
Issue: Whether or Not the seizure of the articles which were not mentioned in the search

warrant was legal.


Held: Section 2 Article III of the 1987 Constitution requires that a search warrant should
particularly describe the things to be seized. The police acts beyond the parameters of
their authority if they seize articles not described in the search warrants. The evident
purpose and intent of the requirement is to limit the things to be seized, to leave the
officers of the law with no discretion; that unreasonable search and seizure may not be
made and that abuses may not be committed.
Petition granted. People of the Philippines is ordered to return the money seized.
f. Placer vs. Villanueva [126 SCRA 463; G.R. Nos. L-60349-62; 29 Dec. 1983]
Facts: Petitioners filed informations in the city court and they certified that Preliminary
Investigation and Examination had been conducted and that prima facie cases have been
found. Upon receipt of said informations, respondent judge set the hearing of the criminal
cases to determine propriety of issuance of warrants of arrest. After the hearing,
respondent issued an order requiring petitioners to submit to the court affidavits of
prosecution witnesses and other documentary evidence in support of the informations to
aid him in the exercise of his power of judicial review of the findings of probable cause
by petitioners. Petitioners petitioned for certiorari and mandamus to compel respondent to
issue warrants of arrest. They contended that the fiscals certification in the informations
of the existence of probable cause constitutes sufficient justification for the judge to issue
warrants of arrest.
Issue: Whether or Not respondent city judge may, for the purpose of issuing warrants of
arrest, compel the fiscal to submit to the court the supporting affidavits and other
documentary evidence presented during the preliminary investigation.
Held: Judge may rely upon the fiscals certification for the existence of probable cause
and on the basis thereof, issue a warrant of arrest. But, such certification does not bind the
judge to come out with the warrant. Theissuance of a warrant is not a mere ministerial
function; it calls for the exercise of judicial discretion on the part of issuing magistrate.
Under Section 6 Rule 112 of the Rules of Court, the judge must satisfy himself of the
existence of probable cause before issuing a warrant of arrest. If on the face of the
information, the judge finds no probable cause, he may disregard the fiscals certification
and require submission of the affidavits of witnesses to aid him in arriving at the
conclusion as to existence of probable cause.
Petition dismissed.

h. Salazar vs. Achacoso [183 SCRA 145; G.R. No. 81510; 14 Mar. 14, 1990]
Facts: Rosalie Tesoro of Pasay City in a sworn statement filed with the POEA, charged
petitioner with illegal recruitment. Public respondent Atty. Ferdinand Marquez sent
petitioner a telegram directing him to appear to the POEA regarding the complaint against
him. On the same day, after knowing that petitioner had no license to operate
a recruitment agency, public respondent Administrator Tomas Achacoso issued a Closure
and Seizure Order No. 1205 to petitioner. It stated that there will a seizure of
thedocuments and paraphernalia being used or intended to be used as the means of
committing illegal recruitment, it having verified that petitioner has (1) No valid
license or authority from the Department of Labor and Employment to recruit and deploy
workers for overseas employment; (2) Committed/are committing acts prohibited under
Article 34 of the New Labor Code in relation to Article 38 of the same code. A team was
then tasked to implement the said Order. The group, accompanied by mediamen and
Mandaluyong policemen, went to petitioners residence. They served the order to a
certain Mrs. For a Salazar, who let them in. The team confiscated assorted costumes.
Petitioner filed with POEA a letter requesting for the return of the seized properties,
because she was not given prior notice and hearing. The said Order violated due process.
She also alleged that it violated sec 2 of the Bill of Rights, and the properties were
confiscated against her will and were done with unreasonable force andintimidation.
Issue: Whether or Not the Philippine Overseas Employment Administration (or the
Secretary of Labor) can validly issue warrants ofsearch and seizure (or arrest) under
Article 38 of the Labor Code
Held: Under the new Constitution, . . . no search warrant or warrant of arrest shall issue
except upon probable cause to be determined personally by the judge
after examination under oath or affirmation of the complainant and the witnesses he may
produce, and particularly describing the place to be searched and the persons or things to
be seized. Mayors and prosecuting officers cannot issue warrants of seizure or arrest.
The Closure and Seizure Order was based on Article 38 of the Labor Code. The Supreme
Court held, We reiterate that the Secretary of Labor, not being a judge, may no longer
issue search or arrest warrants. Hence, the authorities must go through the
judicial process. To that extent, we declare Article 38, paragraph (c), of the Labor Code,
unconstitutional and of no force and effect The power of the President to order the
arrest of aliens for deportation is, obviously, exceptional. It (the power to order arrests)
cannot be made to extend to other cases, like the one at bar. Under the Constitution, it is
the sole domain of the courts. Furthermore, the searchand seizure order was in the nature
of a general warrant. The court held that the warrant is null and void, because it must
identify specifically the things to be seized.
WHEREFORE, the petition is GRANTED. Article 38, paragraph (c) of the Labor Code is
declared UNCONSTITUTIONAL and null and void. The respondents are ORDERED to
return all materials seized as a result of the implementation of Search and Seizure Order
No. 1205.

k. Sales vs. Sandiganbayan [369 SCRA 293 G.R. No. 143802, 16 Nov. 2001]
Facts: The petitioner, the incumbent mayor of Pagudpud Ilocos Norte, shot the former
mayor and his political rival Atty. Benemerito. After the shooting, he surrendered himself
and hence the police inspector and wife of the victim filed a criminal complaint for
murder against him. The judge after conducting the preliminary examination (p.e. for
brevity) found probable cause and issued a warrant of arrest. Also after conducting the
preliminary investigation (p.i. for brevity), he issued a resolution forwarding the case to
the prosecutor for appropriate action. Petitioner received a subpoena directing him
to file his counter affidavit, affidavit of witnesses and other supporting documents. He did
it the following day. While proceedings are ongoing, he filed a petition for habeas corpus
with the C.A alleging that: the warrant was null and void because the judge who issued it
was a relative by affinity of the private respondent and the p.e. and the p.i. were illegal
and irregular as the judge doesnt have jurisdiction on the case. The C.A. granted the
petition holding that the judge was a relative by affinity by 3rd degree to the private
respondent and the p.i. he conducted has 2 stages, the p.e. and the p.i. proper. The
proceeding now consists only of one stage. He conducted the requisite investigation prior
to the issuance of warrant of arrest. Moreover he did not complete it. He only
examined the witness of the complainant. But the prosecution instead of conducting p.i.
of his own forwarded the records to the Ombudsman (OMB for brevity) for the latter to
conduct the same. The OMB directed the petitioner to submit his counteraffidavit, but he
did not comply with it finding the same superfluous. The graft investigator
recommended the filing of information for murder which the OMB approved. Petitioner
received a copy of the resolution but prevented seeking reconsideration thereof he filed a
motion to deferissuance of warrant of arrest pending the determination of probable cause.
The Sandiganbayan denied the motion. This is now a petition for review on the decision
of the Sandiganbayan.
Issues:
(1) Whether or Not the OMB followed the procedure in conducting preliminary
investigation.
(2) Whether or Not petitioner was afforded an opportunity to be heard and to submit
controverting evidence.
Held: The proper procedure in the conduct of preliminary investigation was not followed
because of the following reasons. Firstly, the preliminary investigation was conducted by
3 different investigators, none of whom completed the preliminary investigation There
was not one continuous proceeding but rather, cases of passing the buck, the last one
being the OMB throwing the buck to the Sandiganbayan. Secondly, the charge of murder
is a non bailable offense. The gravity of the offense alone should have merited a deeper
and more thorough preliminary investigation. The OMB did nothing of the sort but

wallowed the resolution of the graft investigator. He did a worse job than the judge, by
actually adopting the resolution of the graft investigator without doing anything and
threw everything to the Sandiganbayan for evaluation. Thirdly, a person under
preliminary investigation by the OMB is entitled to a motion for reconsideration, as
maintained by the Rules of Procedure by the OMB. The filing of the motion for
reconsideration is an integral part of the preliminary investigation proper. The denial
thereof is tantamount to the denial of the right itself to a preliminary investigation. This
fact alone renders preliminary investigation conducted in this case incomplete. And lastly,
it was patent error for the Sandiganbayan to have relied purely on the OMBs certification
of probable cause given the prevailing facts of the case much more so in the face of the
latters flawed report and one side factual findings.
The court cannot accept the Sandiganbayans assertion of having found probable cause on
its own, considering the OMBs defective report and findings, which merely rekied on the
testimonies of the witnesses for the prosecution and disregarded the evidence for the
defense.
Judgment is rendered setting aside the resolution of the Sandiganbayan, ordering the
Sandiganbayan to quash the warrant of arrest and remanding the OMB for completion of
the preliminary investigation.
m. Veroy vs. Layague [210 SCRA 97; G.R. No. 95630; 18 June 1992]
The permission to enter a house and search for persons and effects may be qualified, and
the searching officer may not act in excess of the authority granted to him.
Although the offense of illegal possession of firearms is a malum prohibitum, it does not
follow that the subjects may be seized simply because they are prohibited. A search
warrant is still necessary in the context of this case.
The Veroys moved to QC and left their house in Davao City to a caretaker who
had keys to the kitchen only. The Veroys had the keys to the interior of the house.

Capt. Obrero raided the house based on an information that rebel soldiers are
allegedly hiding there.

With the help of caretakers, they were able to enter only up to the yard since the
owner was not around and they did not have a search warrant.

They contacted Mrs. Veroy, and explained that the house was reportedly being
used as a hideout and recruitment center of rebel soldiers. Mrs. Veroy then gave
permission to search the house with the condition that Major Macasaet, a long-time
family friend, must be there during the search.

Despite the qualified consent, the officers entered various rooms, including the
childrens room, and confiscated a .45 caliber gun and other effects, which were the basis
of the charge of illegal possession of firearms against them.

Despite the fact that the warrants for their arrest have not yet been served on
them, petitioners voluntarily surrendered themselves to Brig. Gen. Pantaleon Dumlao,
PC-CIS Chief, since it was the CIS that initiated the complaint. However, the latter

refused to receive them on the ground that his office has not yet received copies of their
warrants of arrest.

The Spouses Veroy assailed the admissibility of the evidence for being obtained in
violation of their constitutional right against unreasonable search and seizure.
Whether the evidence is admissible? NO.
Petitioners alleged that while Capt. Obrero had permission to enter their house, it
was merely for the purpose of ascertaining the presence of the alleged "rebel" soldiers.
The permission did not include the authority to conduct a room to room search inside the
house. The items taken were, therefore, products of an illegal search, violative of their
constitutional rights. As such, they are inadmissible in evidence against them.

The Court ruled that the case at bar does not fall on the exceptions for a
warrantless search. The reason for searching the house is that it was reportedly being used
as a hideout and recruitment center for rebel soldiers. While Capt. Obrero was able to
enter the yard, he did not enter the house because he did not have a search warrant and
the owners were not present. This shows that he himself recognized the need for a search
warrant, hence, he did not persist in entering the house but rather contacted the Veroys to
seek permission to enter the same. Permission was granted by Mrs. Veroy to enter the
house but only to ascertain the presence of rebel soldiers.

Under the circumstances the police officers had time to procure a search warrant
but they did not.

The Court also ruled that although the offense of illegal possession of firearms is
a malum prohibitum, it does not follow that the subjects may be seized simply because
they are prohibited. A search warrant is still necessary.

The rule having been violated and no exception being applicable, the articles
seized were confiscated illegally and are therefore protected by the exclusionary
principle. They cannot be used as evidence against the petitioners in the criminal action
against them for illegal possession of firearms.

o. Pp vs. Gesmundo [219 SCRA 743; G.R. No. 89373; 19 Mar. 1993]
Facts: According to the prosecution, in the morning of Nov. 17, 1986, PO Jose Luciano
gave money and instructed his civilian informer to buy marijuana from the accused at the
Cocoland Hotel. He actually saw the accused selling marijuana to his
civilian informer and that same day Luciano applied for a search warrant.
About 2pm that day, a police raiding team armed with a search warrant went to the Brgy
captain for them to be accompanied in serving the said warrant at the residence of the
accused. The police was allowed to enter the house upon the strength of the warrant
shown to the accused. The accused begged the police not to search and to leave the house.
The police still searched the house and was led to the kitchen. She pointed a metal basin
on top of a table as the hiding place of died marijuana flowering tops contained in a
plastic bag marked ISETANN. The police also recovered from a native uway cabinet
dried marijuana flowering tops wrapped in 3 pieces of komiks paper.
According to the accused, when the police arrived at her house, she saw Sgt. Yte and PFC

Jose Luciano. She invited Sgt. Yte to enter her house while Luciano was left in the jeep
that was parked near the house. While inside the house Yte showed the accused
something he claimed as a searchwarrant, when someone coming from the kitchen uttered
eto na They proceeded to the kitchen and saw Luciano holding a plastic bag with four
other companions. They confronted the accused and insisted that the bags belonged to
her. Accused denied the accusation and told them that she doesnt know anything about it.
She was made to sign a prepareddocument. She was brought to the police station and was
detained.
The court renders judgment finding the accused guilty.
Issue: Whether or Not the evidence was properly obtained by the police.
Held: In the investigation report prepared by Luciano stated that during the search they
discovered a hole at the backyard of the house of the suspect, there was a big biscuit can
inside the hole and on top of the cover a flower pot was placed wherein the marijuana
was kept. However, there was no mention of any marijuana obtained from a flower pot in
any of their testimonies. There were inconsistencies insofar the prosecution is concerned,
as to what was recovered and where, the trial court concluded that these inconsistencies
are trivial. There must sufficient evidence that the marijuana was actually surrendered by
the accused. As held in PP vs. Remorosa, Irreconcilable and
unexplained contradictions in the testimonies of the prosecution witnesses cast doubt on
the guilt of appellant and his culpability to the crime charged.
The claim that the marijuana was planted was strengthen as the police violated sec 7, rule
126 rules of the court provides no search of a house, room or any other premise shall be
made except in the presence of the lawful occupant thereof or any member of his family
or in the absence of the latter, in the presence of two (2) witnesses of sufficient age and
discretion residing in the same locality. This requirement is mandatory to ensure
regularity in the execution of the search warrant. Violation of said rule is in fact
punishable under Article 130 of the Revised Penal Code.
The document (PAGPAPATUNAY) was inadmissible to the court as the accused was not
informed of her right not to sign the document neither was she informed that she has the
right to the assistance of a counsel and the fact that it may be used as evidence against
her. It was not proved that the marijuana belonged to her. Not only does the law require
the presence of witnesses when the search is conducted, but it also imposes upon the
person making the search the duty to issue a detailed receipt for the property seized. He is
likewise required to deliver the property seized to the judge who issued the warrant,
together with a true and accurate inventory thereof duly verified under oath. Again, these
duties are mandatory and are required to preclude substitution of the items seized
by interested parties.
The guilt of the accused was has not been established. Judgment is reversed.

s. Pp vs. Sy Chua [ 396 SCRA 657; G.R. No.136066-67, 4 February 2003]


Facts: Accused-appellant Binad Sy Chua was charged with violation of Section 16,
Article III of R.A. 6425, as amended by R.A. 7659, and for Illegal Possession of
Ammunitions and Illegal Possession of Drugs in two separate Informations.
SPO2 Nulud and PO2 Nunag received a report from their confidential informant that
accused-appellant was about to deliver drugs that night at the Thunder Inn Hotel in
Balibago, Angeles City. So, the PNP Chief formed a team of operatives. The group
positioned themselves across McArthur Highway near Bali Hai Restaurant, fronting the
hotel. The other group acted as their back up.
Afterwards, their informer pointed to a car driven by accused-appellant which just arrived
and parked near the entrance of the hotel. After accused-appellant alighted from the car
carrying a sealed Zest-O juice box, SPO2 Nulud and PO2 Nunag hurriedly accosted him
and introduced themselves as police officers. As accused-appellant pulled out his wallet, a
small transparent plastic bag with a crystalline substance protruded from his right back
pocket. Forthwith, SPO2 Nulud subjected him to a body search which yielded twenty
(20) pieces of live .22 caliber firearm bullets from his left back pocket. When SPO2
Nunag peeked into the contents of the Zest-O box, he saw that it contained a
crystalline substance. SPO2 Nulud instantly confiscated the small transparent plastic bag,
the Zest-O juice box, the twenty (20) pieces of .22 caliber firearm bullets and the car used
by accused-appellant. SPO2 Nulud and the other police operatives who arrived at the
scene brought the confiscated items to the office of Col. Guttierez at the PNP
Headquarters in Camp Pepito, Angeles City.
Accused-appellant vehemently denied the accusation against him and narrated a
different version of the incident.
Accused-appellant alleged that he was driving the car of his wife to follow her and his
son to Manila. He felt sleepy, so he decided to take the old route along McArthur
Highway. He stopped in front of a small store near Thunder Inn Hotel to
buy cigarettes and candies. While at the store, he noticed a man approaches and examines
the inside of his car. When he called the attention of the onlooker, the man immediately
pulled out a .45 caliber gun and made him face his car with raised hands. The man later
on identified himself as a policeman. During the course of the arrest, the policeman took
out his wallet and instructed him to open his car. He refused, so the policeman took his
car keys and proceeded to search his car. At this time, the police officers companions
arrived at the scene in two cars. PO2 Nulud, who just arrived at the scene, pulled him
away from his car in a nearby bank, while the others searched his car.
Thereafter, he was brought to a police station and was held inside a bathroom for about
fifteen minutes until Col. Guttierez arrived, who ordered his men to call the media. In the
presence of reporters, Col. Guttierez opened the box and accused-appellant was made to
hold the box whilepictures were being taken.

The lower court acquitted Sy Chua for the Illegal Possession of Ammunitions, yet
convicted him for Illegal Possession of 1,955.815 grams of shabu. Hence, this appeal to
the Court.
Issues:
(1) Whether or Not the arrest of accused-appellant was lawful; and
(2) WON the search of his person and the subsequent confiscation of shabu allegedly
found on him were conducted in a lawful and valid manner.
Held: The lower court believed that since the police received information that the
accused will distribute illegal drugs that evening at the Thunder Inn Hotel and its
vicinities. The police officer had to act quickly and there was no more time to secure
a search warrant. The search is valid being akin to a stop and frisk.
The trial court confused the concepts of a stop-and-frisk and of a searchincidental to a
lawful arrest. These two types of warrantless searches differ in terms of the requisite
quantum of proof before they may be validly effected and in their allowable scope.
In a search incidental to a lawful arrest, as the precedent arrest determines the validity of
the incidental search, the legality of the arrest is questioned, e.g., whether an arrest was
merely used as a pretext for conducting asearch. In this instance, the law requires that
there first be arrest before asearch can be madethe process cannot be reversed.
Accordingly, for this exception to apply, two elements must concur: (1) the person to be
arrested must execute an overt act indicating that he has just committed, is actually
committing, or is attempting to commit a crime; and (2) such overt act is done in the
presence or within the view of the arresting officer.
We find the two aforementioned elements lacking in the case at bar. Accused-appellant
did not act in a suspicious manner. For all intents and purposes, there was no overt
manifestation that accused-appellant has just committed, is actually committing, or is
attempting to commit a crime. Reliable information alone, absent any overt act
indicative of a felonious enterprise in the presence and within the view of the arresting
officers, is not sufficient to constitute probable cause that would justify an in flagrante
delicto arrest.
With regard to the concept of stop-and frisk: mere suspicion or a hunch will not
validate a stop-and-frisk. A genuine reason must exist, in light of the police officers
experience and surrounding conditions, to warrant the belief that the person detained has
weapons concealed about him. Finally, a stop-and-frisk serves a two-fold interest: (1)
the general interest of effective crime prevention and detection for purposes of
investigating possible criminal behavior even without probable cause; and (2) the interest
of safety and self-preservation which permit the police officer to take steps to assure
himself that the person with whom he deals is not armed with a deadly weapon that could

unexpectedly and fatally be used against the police officer.


A stop-and-frisk was defined as the act of a police officer to stop a citizen on the street,
interrogate him, and pat him for weapon(s) or contraband. It should also be emphasized
that a search and seizure should precede the arrest for this principle to apply. The
foregoing circumstances do not obtain in the case at bar. To reiterate, accused-appellant
was first arrested before the search and seizure of the alleged illegal items found in his
possession. The apprehending police operative failed to make any initial inquiry into
accused-appellants business in the vicinity or the contents of the Zest-O juice box he was
carrying. The apprehending police officers only introduced themselves when they already
had custody of accused-appellant.
In the case at bar, neither the in flagrante delicto nor the stop and frisk principles is
applicable to justify the warrantless arrest and consequentsearch and seizure made by the
police operatives on accused-appellant.
Wherefore, accused-appellant Binad Sy Chua is hereby Acquitted.
u. Callanta vs. Villanueva [77 SCRA 377; G.R. Nos. 24646 & L-24674; 20 June 1977]
Facts: Two complaints for grave oral defamation were filed against Faustina Callanta.
The City Judge of Dagupan City, Felipe Villanueva, denied the motions to quash the
complaints. Thus, petitioner Callanta brought the suits for certiorari in the Supreme
Court. Petitioner questions the validity of the issuance of warrant of arrest by respondent,
arguing that the City Fiscal should have conducted the preliminary investigation.
According to petitioners counsel, there was jurisdictional infirmity. After the issuance of
the warrants of arrest and the bail fixed at P600, petitioner posted the bail bond, thus
obtaining her provisional liberty. The City Fiscal in this case did not disagree with the
judges investigation, and agreed with the complaints filed.
Issue: Whether or Not petitioners contentions are to be given merit.
Held: Based on many precedent cases of the Supreme Court, where the accused has
filed bail and waived the preliminary investigation proper, he has waived
whatever defect, if any, in the preliminary examinationconducted prior to the issuance of
the warrant of arrest. In the case at bar, it is futile for the petitioner to question the
validity of the issuance of the warrant of arrest, because she posted the bail bond.
Petitioner also erred in arguing that only the City Fiscal can conduct a preliminary
investigation. According to the Charter of the City of Dagupan, the City Court of
Dagupan City may also conduct preliminary investigation for any offense, without regard
to the limits of punishment, and may release, or commit and bindover any person charged
with such offense to secure his appearance before the proper court. Petition for certiorari
is denied. Restraining order issued by the Court is lifted and set aside.

v. Posadas vs. CA [188 SCRA 288; G.R. No. 89139; 2 Aug. 1990]
POSADAS VS. COURT OF APPEALS [188 SCRA 288; G.R. NO. 89139; 2 AUG 1990]
Facts:
Members of the Integrated National Police (INP) of the Davao Metrodiscom assigned
with the Intelligence Task Force, Pat. Ursicio Ungab and Pat. Umbra Umpar conducted
surveillance along Magallanes Street, Davao City. While in the vicinity of Rizal
Memorial Colleges they spotted petitioner carrying a "buri" bag and they noticed him to
be acting suspiciously. They approached the petitioner and identified themselves as
members of the INP. Petitioner attempted to flee but his attempt to get away was
unsuccessful. They then checked the "buri" bag of the petitioner where they found one (1)
caliber .38 Smith & Wesson revolver with Serial No. 770196, two (2) rounds of live
ammunition for a .38 caliber gun, a smoke (tear gas) grenade, and two (2) live
ammunitions for a .22 caliber gun. They brought the petitioner to the police station for
further investigation. In the course of the same, the petitioner was asked to show the
necessary license or authority to possess firearms and ammunitions found in his
possession but he failed to do so. He was then taken to the Davao Metrodiscom office and
the prohibited articles recovered from him were indorsed to M/Sgt. Didoy the officer then
on duty. He was prosecuted for illegal possession of firearms and ammunitions in the
Regional Trial Court of Davao City.
Issue:
Whether or Not the warantless search is valid.
Held:
In justifying the warrantless search of the buri bag then carried by the petitioner, argues
that under Section 12, Rule 136 of the Rules of Court a person lawfully arrested may be
searched for dangerous weapons or anything used as proof of a commission of an offense
without a search warrant. It is further alleged that the arrest without a warrant of the
petitioner was lawful under the circumstances. in the case at bar, there is no question that,
indeed, it is reasonable considering that it was effected on the basis of a probable cause.
The probable cause is that when the petitioner acted suspiciously and attempted to flee
with the buri bag there was a probable cause that he was concealing something illegal in
the bag and it was the right and duty of the police officers to inspect the same. It is too
much indeed to require the police officers to search the bag in the possession of the
petitioner only after they shall have obtained a search warrant for the purpose. Such an
exercise may prove to be useless, futile and much too late. Clearly, the search in the case
at bar can be sustained under the exceptions heretofore discussed, and hence, the
constitutional guarantee against unreasonable searches and seizures has not been violated.
w. Pp vs. Mengote [210 SCRA 174; G.R. No. 87059; 22 June 1992]
Facts: The Western Police District received a telephone call from aninformer that there
were three suspicious looking persons at the corner of Juan Luna and North
Bay Boulevard in Tondo, Manila. A surveillance team of plainclothesmen was forthwith

dispatched to the place. The patrolmen saw two men looking from side to side, one of
whom holding his abdomen. They approached the persons and identified themselves as
policemen, whereupon the two tried to run but unable to escape because the other lawmen
surrounded them. The suspects were then searched. One of them the accused-appellant
was found with a .38 caliber with live ammunitions in it, while his companion had a fan
knife. The weapons were taken from them and they were turned over to the police
headquarters for investigation. An information was filed before the RTC convicting the
accused of illegal possession of firearm arm. A witness testified that the weapon was
among the articles stolen at his shop, which he reported to the police including the
revolver. For his part, Mengote made no effort to prove that he owned the fire arm or that
he was licensed to possess it but instead, he claimed that the weapon was planted on him
at the time of his arrest. He was convicted for violation of P.D.1866 and was sentenced to
reclusion perpetua. In his appeal he pleads that the weapon was not admissible as
evidence against him because it had been illegally seized and therefore the fruit of
apoisonous tree.
Issue: Whether or not the warrantless search and arrest was illegal.
Held: An evidence obtained as a result of an illegal search and seizure inadmissible in
any proceeding for any purpose as provided by Art. III sec 32 of the Constitution. Rule
113 sec.5 of the Rules of Court, provides arrest without warrant lawful when: (a) the
person to be arrested has committed, is actually committing, or is attempting to commit
an offense, (b) when the offense in fact has just been committed, and he has
personal knowledge ofthe facts indicating the person arrested has committed it and (c) the
person to be arrested has escaped from a penal establishment or a place where he is
serving final judgment or temporarily confined while his case is pending, or has escaped
while being transferred from one confinement to another.
These requirements have not been established in the case at bar. At the time of the arrest
in question, the accused appellant was merely looking from side to side and holding his
abdomen, according to the arresting officers themselves. There was apparently no offense
that has just been committed or was being actually committed or at least being attempt by
Mengote in their presence. Moreover a person may not be stopped and frisked in a
broad daylight or on a busy street on unexplained suspicion.
Judgment is reversed and set aside. Accused-appellant is acquitted.
x. Pp vs. Tangliben [184 SCRA 220; G.R. No.L-63630; 6 Apr. 1990]
Facts: Patrolmen Silverio and Romeo Punzalan were conducting surveillance at the San
Fernando Victory Liner Terminal. At around 9:30pm they noticed a person, Medel
Tangliben, carrying a traveling bag who acted suspiciously. They confronted him,
inspected his bag, and there they foundmarijuana leaves. The accused was then taken
to the Police Headquarters for further investigations. The TC found Tangliben guilty of
violating sec.4 art. 2 of the RA 6425 or the Dangerous Drugs Act of 1972.

Issue: Whether or Not there was an unlawful search due to lack of search warrant.
Held: No. Rule 113 sec. 5 provides the a peace officer or a private person may w/o a
warrant arrest a person when in his presence the person to be arrested has committed, is
committing, or is attempting to commit an offense.
In the present case, the accused was found to have been committing possession
of marijuana and can be therefore searched lawfully even without a search warrant.
Another reason is that this case poses urgency on the part of the arresting police officers.
It was found out that an informer pointed to the accused telling the policemen that the
accused was carryingmarijuana. The police officers had to act quickly and there was not
enough time to secure a search warrant.
aa. Pp vs. Saycon [236 SCRA 325; G.R. No. 110995; 5 Sept. 1994]
Facts: On or about 8 July 1992, at about 6:00 in the morning, the Coastguard personnel
received information from NARCOM agent Ruben Laddaran that a suspected "shabu"
courier by the name of Alvaro Saycon was on board the MV Doa Virginia, which was
arriving at that moment in Dumaguete City. Upon receipt of the information, the
Coastguard chief officer CPO Tolin, instructed them to intercept the suspect. A combined
team of NARCOM agents and Philippine Coastguard personnel consisting of CPO Tolin,
a certain Miagme, and Senior Police Officers Ruben Laddaran and Winifredo Noble of
NARCOM posted themselves at the gate of Pier 1. The MV Doa Virginia docked at 6:00
a.m. that same morning at Pier 1 in Dumaguete City. Alvaro Saycon alighted from the
boat carrying a black bag and went through the checkpoint manned by the Philippine
Coastguard where he was identified by police officer Winifredo Noble of NARCOM.
Saycon was then invited to the Coastguard Headquarters at the Pier area. He willingly
went with them. At the headquarters, the coastguard asked Saycon to open his bag, and
the latter willingly obliged. In it were personal belongings and a maong wallet. Inside that
maong wallet, there was a Marlboro pack containing the suspected "shabu". When police
officer Winifredo Noble asked Saycon whether the Marlboro pack containing the
suspected "shabu" was his, Saycon merely bowed his head. Then Saycon, his bag and the
suspected "shabu" were brought to the NARCOM office for booking. When Alvaro
Saycon was arrested, the NARCOM agents did not have a warrant of arrest. The PNP's
Forensic Analyst declared in court that she had conducted an examination of the
specimens and found out that the specimens weighed 4.2 grams in total, consisted of
methamphetamine hydrochloride, more widely known as "shabu."
Issue: Whether or Not the warrantless search was valid.
Held: The warrantless search was valid, as the accused was a passenger of a motor
vehicle. There was probable cause to believe that the accused was carrying prohibited

drugs. Three weeks earlier, agents of the Narcotics Command bought methamine
hydrochloride from him. An agent of the Narcotics Command reported that the accused
would be arriving on board the vessel and carrying methamphetamine hydrochloride with
him. Drug couriers do not go about their trade with some external sign indicating that
they are transporting prohibited drugs. This must be taken into account in determining
probable cause.
Pita vs. CA [178 SCRA 362; G.R. No. 80806; 5 Oct. 1989]
Facts: On December 1 and 3, 1983, pursuing an Anti-Smut Campaign initiated by the
Mayor of the City of Manila, Ramon D. Bagatsing, elements of the Special AntiNarcotics Group, Auxilliary Services Bureau, Western Police District, INP of the
Metropolitan Police Force of Manila, seized and confiscated from dealers, distributors,
newsstand owners and peddlers along Manila sidewalks, magazines, publications and
other reading materials believed to be obscene, pornographic and indecent and later
burned the seized materials in public at the University belt along C.M. Recto Avenue,
Manila, in the presence of Mayor Bagatsing and several officers and members of various
student organizations.
Among the publications seized, and later burned, was "Pinoy Playboy" magazines
published and co-edited by plaintiff Leo Pita.
Plaintiff filed a case for injunction with prayer for issuance of the writ of preliminary
injunction against Mayor Bagatsing and Narcisco Cabrera, as superintendent of Western
Police District of the City of Manila, seeking to enjoin said defendants and their agents
from confiscating plaintiffs magazines or from preventing the sale or circulation thereof
claiming that the magazine is a decent, artistic and educational magazine which is not per
se obscene, and that the publication is protected by the Constitutional guarantees of
freedom of speech and of the press. Plaintiff also filed an Urgent Motion for issuance of a
temporary restraining order against indiscriminate seizure, confiscation and burning of
plaintiff's "Pinoy Playboy" Magazines, pending hearing on the petition for preliminary
injunction. The Court granted the temporary restraining order. The case was set for trial
upon the lapse of the TRO. RTC ruled that the seizure was valid. This was affirmed by
the CA.
Issue: Whether or Not the seizure violative of the freedom of expression of the petitioner.
Held: Freedom of the press is not without restraint as the state has the right to protect
society from pornographic literature that is offensive to public morals, as indeed we have
laws punishing the author, publishers and sellers of obscene publications. However, It is
easier said than done to say, that if the pictures here in question were used not exactly for
art's sake but rather for commercial purposes, the pictures are not entitled to any
constitutional protection. Using the Kottinger rule: the test of obscenity is "whether the
tendency of the matter charged as obscene, is to deprave or corrupt those whose minds

are open to such immoral influences and into whose hands a publication or other article
charged as being obscene may fall." Another is whether it shocks the ordinary and
common sense of men as an indecency. Ultimately "whether a picture is obscene or
indecent must depend upon the circumstances of the case and that the question is to be
decided by the "judgment of the aggregate sense of the community reached by it." The
government authorities in the instant case have not shown the required proof to justify a
ban and to warrant confiscation of the literature First of all, they were not possessed of a
lawful court order: (1) finding the said materials to be pornography, and (2) authorizing
them to carry out a search and seizure, by way of a search warrant. The court provides
that the authorities must apply for the issuance of a search warrant from a judge, if in
their opinion an obscenity seizure is in order and that;
1. The authorities must convince the court that the materials sought to be seized are
obscene and pose a clear and present danger of an evil substantive enough to warrant
State interference and action;
2. The judge must determine whether or not the same are indeed obscene. The question is
to be resolved on a case-to-case basis and on the judges sound discretion;
Guanzon vs. De Villa [181 SCRA 623; G.R. 80508; 30 Jan. 1990]
Facts: The 41 petitioners alleged that the "saturation drive" or "aerial target zoning" that
were conducted in their place (Tondo Manila) were unconstitutional. They alleged that
there is no specific target house to be search and that there is no search warrant or warrant
of arrest served. Most of the policemen are in their civilian clothes and without
nameplates or identification cards. The residents were rudely rouse from their sleep by
banging on the walls and windows of their houses. The residents were at the point of
high-powered guns and herded like cows. Men were ordered to strip down to their briefs
for the police to examine their tattoo marks. Theresidents complained that they're homes
were ransacked, tossing their belongings and destroying their valuables. Some of their
money andvaluables had disappeared after the operation.
The residents also reportedincidents of maulings, spot-beatings and maltreatment. Those
who were detained also suffered mental and physical torture to extract confessions and
tactical informations. The respondents said that such accusations were all lies.
Respondents contends that the Constitution grants to government the power to seek and
cripple subversive movements for the maintenance of peace in the state. The aerial target
zoning were intended to flush out subversives and criminal elements coddled by
the communities were the said drives were conducted. They said that they have
intelligently and carefully planned months ahead for the actual operation and that local
and foreign media joined the operation to witness and record such event.
Issue: Whether or Not the saturation drive committed consisted of violation of human
rights.
Held: It is not the police action per se which should be prohibited rather it is the
procedure used or the methods which "offend even hardened sensibilities" .Based on the

facts stated by the parties, it appears to have been no impediment to securing search
warrants or warrants of arrest before any houses were searched or individuals roused
from sleep were arrested. There is no showing that the objectives sought to be attained by
the "aerial zoning" could not be achieved even as th rights of the squatters and low
income families are fully protected. However, the remedy should not be brought by a
tazpaer suit where not one victim complaints and not one violator is properly charged. In
the circumstances of this taxpayers' suit, there is no erring soldier or policeman whom the
court can order prosecuted. In the absence of clear facts no permanent relief can be given.
In the meantime where there is showing that some abuses were committed, the court
temporary restraint the alleged violations which are shocking to the senses. Petition is
remanded to the RTC of Manila.

Rule 113 and 126 of the Rules of Court


RULE 113
Arrest
Section 1. Definition of arrest. Arrest is the taking of a person into custody in order
that he may be bound to answer for the commission of an offense. (1)
Section 2. Arrest; how made. An arrest is made by an actual restraint of a person to be
arrested, or by his submission to the custody of the person making the arrest.
No violence or unnecessary force shall be used in making an arrest. The person arrested
shall not be subject to a greater restraint than is necessary for his detention. (2a)
Section 3. Duty of arresting officer. It shall be the duty of the officer executing the
warrant to arrest the accused and to deliver him to the nearest police station or jail
without unnecessary delay. (3a)
Section 4. Execution of warrant. The head of the office to whom the warrant of arrest
was delivered for execution shall cause the warrant to be executed within ten (10) days
from its receipt. Within ten (10) days after the expiration of the period, the officer to
whom it was assigned for execution shall make a report to the judge who issued the
warrant. In case of his failure to execute the warrant, he shall state the reasons therefor.
(4a)
Section 5. Arrest without warrant; when lawful. A peace officer or a private person
may, without a warrant, arrest a person:
(a) When, in his presence, the person to be arrested has committed, is actually
committing, or is attempting to commit an offense;

(b) When an offense has just been committed, and he has probable cause to believe based
on personal knowledge of facts or circumstances that the person to be arrested has
committed it; and
(c) When the person to be arrested is a prisoner who has escaped from a penal
establishment or place where he is serving final judgment or is temporarily confined
while his case is pending, or has escaped while being transferred from one confinement to
another.
In cases falling under paragraph (a) and (b) above, the person arrested without a warrant
shall be forthwith delivered to the nearest police station or jail and shall be proceeded
against in accordance with section 7 of Rule 112. (5a)
Section 6. Time of making arrest. An arrest may be made on any day and at any time
of the day or night. (6)
Section 7. Method of arrest by officer by virtue of warrant. When making an arrest by
virtue of a warrant, the officer shall inform the person to be arrested of the cause of the
arrest and of the fact that a warrant has been issued for his arrest, except when he flees or
forcibly resists before the officer has opportunity to so inform him, or when the giving of
such information will imperil the arrest. The officer need not have the warrant in his
possession at the time of the arrest but after the arrest, if the person arrested so requires,
the warrant shall be shown to him as soon as practicable. (7a)
Section 8. Method of arrest by officer without warrant. When making an arrest
without a warrant, the officer shall inform the person to be arrested of his authority and
the cause of the arrest, unless the latter is either engaged in the commission of an offense,
is pursued immediately after its commission, has escaped, flees or forcibly resists before
the officer has opportunity so to inform him, or when the giving of such information will
imperil the arrest. (8a)
Section 9. Method of arrest by private person. When making an arrest, a private
person shall inform the person to be arrested of the intention to arrest him and cause of
the arrest, unless the latter is either engaged in the commission of an offense, is pursued
immediately after its commission, or has escaped, flees, or forcibly resists before the
person making the arrest has opportunity to so inform him, or when the giving of such
information will imperil the arrest. (9a)
Section 10. Officer may summon assistance. An officer making a lawful arrest may
orally summon as many persons as he deems necessary to assist him in effecting the
arrest. Every person so summoned by an officer shall assist him in effecting the arrest
when he can render such assistance without detriment to himself. (10a)
Section 11. Right of officer to break into building or enclosure. An officer, in order to
make an arrest either by virtue of a warrant, or without a warrant as provided in section 5,
may break into any building or enclosure where the person to be arrested is or is

reasonably believed to be, if he is refused admittance thereto, after announcing his


authority and purpose. (11a)
Section 12. Right to break out from building or enclosure. Whenever an officer has
entered the building or enclosure in accordance with the preceding section, he may break
out therefrom when necessary to liberate himself. (12a)
Section 13. Arrest after escape or rescue. If a person lawfully arrested escapes or is
rescued, any person may immediately pursue or retake him without a warrant at any time
and in any place within the Philippines. (13)
Section 14. Right of attorney or relative to visit person arrested. Any member of the
Philippine Bar shall, at the request of the person arrested or of another acting in his
behalf, have the right to visit and confer privately with such person in the jail or any other
place of custody at any hour of the day or night. Subject to reasonable regulations, a
relative of the person arrested can also exercise the same right. (14a)

RULE 126
Search and Seizure
Section 1. Search warrant defined. A search warrant is an order in writing issued in
the name of the People of the Philippines, signed by a judge and directed to a peace
officer, commanding him to search for personal property described therein and bring it
before the court. (1)
Section 2. Court where application for search warrant shall be filed. An application
for search warrant shall be filed with the following:
a) Any court within whose territorial jurisdiction a crime was committed.
b) For compelling reasons stated in the application, any court within the judicial region
where the crime was committed if the place of the commission of the crime is known, or
any court within the judicial region where the warrant shall be enforced.
However, if the criminal action has already been filed, the application shall only be made
in the court where the criminal action is pending. (n)
Section 3. Personal property to be seized. A search warrant may be issued for the
search and seizure of personal property:
(a) Subject of the offense;

(b) Stolen or embezzled and other proceeds, or fruits of the offense; or


(c) Used or intended to be used as the means of committing an offense. (2a)
Section 4. Requisites for issuing search warrant. A search warrant shall not issue
except upon probable cause in connection with one specific offense to be determined
personally by the judge after examination under oath or affirmation of the complainant
and the witnesses he may produce, and particularly describing the place to be searched
and the things to be seized which may be anywhere in the Philippines. (3a)
Section 5. Examination of complainant; record. The judge must, before issuing the
warrant, personally examine in the form of searching questions and answers, in writing
and under oath, the complainant and the witnesses he may produce on facts personally
known to them and attach to the record their sworn statements, together with the
affidavits submitted. (4a)
Section 6. Issuance and form of search warrant. If the judge is satisfied of the
existence of facts upon which the application is based or that there is probable cause to
believe that they exist, he shall issue the warrant, which must be substantially in the form
prescribed by these Rules. (5a)
Section 7. Right to break door or window to effect search. The officer, if refused
admittance to the place of directed search after giving notice of his purpose and authority,
may break open any outer or inner door or window of a house or any part of a house or
anything therein to execute the warrant or liberate himself or any person lawfully aiding
him when unlawfully detained therein. (6)
Section 8. Search of house, room, or premise to be made in presence of two witnesses.
No search of a house, room, or any other premise shall be made except in the presence of
the lawful occupant thereof or any member of his family or in the absence of the latter,
two witnesses of sufficient age and discretion residing in the same locality. (7a)
Section 9. Time of making search. The warrant must direct that it be served in the day
time, unless the affidavit asserts that the property is on the person or in the place ordered
to be searched, in which case a direction may be inserted that it be served at any time of
the day or night. (8)
Section 10. Validity of search warrant. A search warrant shall be valid for ten (10)
days from its date. Thereafter it shall be void. (9a)
Section 11. Receipt for the property seized. The officer seizing property under the
warrant must give a detailed receipt for the same to the lawful occupant of the premises
in whose presence the search and seizure were made, or in the absence of such occupant,
must, in the presence of at least two witnesses of sufficient age and discretion residing in
the same locality, leave a receipt in the place in which he found the seized property. (10a)

Section 12. Delivery of property and inventory thereof to court; return and proceedings
thereon. (a) The officer must forthwith deliver the property seized to the judge who
issued the warrant, together with a true inventory thereof duly verified under oath.
(b) Ten (10) days after issuance of the search warrant, the issuing judge shall ascertain if
the return has been made, and if none, shall summon the person to whom the warrant was
issued and require him to explain why no return was made. If the return has been made,
the judge shall ascertain whether section 11 of this Rule has been complained with and
shall require that the property seized be delivered to him. The judge shall see to it that
subsection (a) hereof has been complied with.
(c) The return on the search warrant shall be filed and kept by the custodian of the log
book on search warrants who shall enter therein the date of the return, the result, and
other actions of the judge.
A violation of this section shall constitute contempt of court.(11a)
Section 13. Search incident to lawful arrest. A person lawfully arrested may be
searched for dangerous weapons or anything which may have been used or constitute
proof in the commission of an offense without a search warrant. (12a)
Section 14. Motion to quash a search warrant or to suppress evidence; where to file. A
motion to quash a search warrant and/or to suppress evidence obtained thereby may be
filed in and acted upon only by the court where the action has been instituted. If no
criminal action has been instituted, the motion may be filed in and resolved by the court
that issued the search warrant. However, if such court failed to resolve the motion and a
criminal case is subsequent filed in another court, the motion shall be resolved by the
latter court. (n)

Você também pode gostar